Sunteți pe pagina 1din 65

Dental management of patients with medical conditions:

Clinical summary – 2017

Information based of:


-Therapeutic guidelines
-MED notes
-HAM notes
Davids notes
- Oxford clinical handbook

Table of contents:

Cardiovascular conditions
Hematological conditions
Respiratory conditions
Endocrine disorders
Gastrointestinal disorders
Musculoskeletal disorders
Renal diseases
Psychiatric disorders
Allergies
Occular emergencies
Basic life support
HISTORY OF GENERAL ANAESTHETIC / HOSPITAL STAY

Question 1: Have you ever stayed in hospital, had an operation or general anaesthetic?

 Questions to ask:
-- Reason for the stay? -- How long ago was it? -- Everything ok now?

 Relays information about overall health and recent/past medical compromisation of the pt
o May impact present OH
o Does pt need referral to medical GP?
o Does GP need to be consulted prior to tx?

 Dental implications
o Some influence tx planning
 Defer elective tx for 3 months after:
 Myocardial infarction
 Coronary bypass surgery
 Stent/ prosthetic valve placement
 Transplant surgeries
 Joint replacement surgeries
 AB prophylaxis considered
 LA may be C/I
 <6 weeks of myocardial infarction
o Patient medications/ immunocompromised
 Antiplatelets/ anticoagulants   bleeding
 Cyclosporine  gingival hyperplasia, interacts with rifampicin

HISTORY OF SERIOUS PROBLEMS AFTER DENTAL TREATMENT?


Question 3: Have you ever had any serious problems after dental treatment?
 Questions to ask
-- What happened? (possible predisposition) -- How long ago? -- Problem resolved?

 Common Issues
 Allergies - LA, latex, penicillin (3 most common)
o Ask about symptoms and extent (urticaria /anaphylaxis)
o Refer to GP – to test for true allergy
o Prevention of overnight symptom precipitation  early appointment

 Post-operative sensitivity
o Predisposition? Choose to line dentine where this is an option

 Swelling and pain post extraction


o Consider NSAIDs prior extraction

 Alveolar osteitis (dry socket)


o Clot disintegration exposing the alveolar bone lining (lamina dura) to the oral
environment (3-5 days post op)
o Clinical Features:
 Bone visible in sockets | Strong dull trobbing pain | Halitosis
o Management:
 Irrigate socket with warm saline solution to remove debris and bacteria
 Analgesics/Systemic Antibiotics
 Socket dressing with Alvogyl  pain relief but delays healing
 Contains:
 Butamben – anaesthetic
 Iodoform - antimicrobial
 Eugenol – analgesic
o Prevention:
 CHX mouthwash on day and few days after surgery
 AB prophylaxis for immunocompromised patient (??)
 Identify high risk pt. beforehand
 Smoker
 Females on oral contraceptives
 Good post op instructions  AVOID smoking

 Osteomyelitis
o Infection of bone generally caused by bacteria
o Delay definitive tx until infection subsides
o Types:
 Acute – generally suppurative, early stage of the disease
 Chronic – present for >1 month (suppurative or non-suppurative)
o Clinical Features:
 Fever | Tenderness and swelling
o Management:
 Surgical debridement
 Long term AB

 Severe bleeding -- Underlying bleeding disease / susceptibility


o Identify cause:
 Anticoagulant/antiplatelet therapy | Surgical trauma
o Precautions:
 Suctioning / Vasocostrictor in LA / Topical haemostatic agent / Suture

 BRONJ (Osteonecrosis of the jaw in pt. taking bisphosphonates)


o Clinical Features:
 Pain | Exposed bone in oral cavity | Draining sinus tract | Soft
tissue infection

 Osteoradionecrosis

 Surgical site infection


o Consider antibiotics if the pt. is immunocompromised

 Dental anxiety
o Consider techniques to overcome anxiety (conservative / sedation)
o Empathize, normalize and guide pt. through tx
 Relaxation
 Imagination
 Distraction
Question 4: Have you ever had any type of heart disease,
CARDIOVASCULAR CONDITIONS: heart murmur, high blood pressure, or rheumatic fever?

Brief overview of cardiac anatomy

Venous returns from systemic circuit


-> Right atrium deoxygenated
-> Right ventricle
-> Pulmonary system
-> Left atrium oxygenated
-> Left ventricle
-> Systemic circuit

Coronary artery disease: leading cause of death Caused by numerous risk factors:
 Presenting as: - Hypetention
o Acute myocardial infarction - Hyperlipidemia
o Angina - Diabetes
o Arrhythmia - Smoking
o Congestive heart failure - Family hx
- Others: obesity, sedentary lifestyle,
Atherosclerosis (accumulation of lipid plaques in the artery kidney disease (influence on
walls) leads to these presentations: hypertension and hyperlipidemia)
- Formation: Damage to wall, commences
inflammatory response, cholesterol pushed into layers of artery, builds in size occluding vessels, cap may
rupue triggering inflammatory cascade and clotting, may completely occlude vessel or clot may dislodge 
MI/Stroke

Ischaemic heart disease: decreased blood supply to part of the heart by narrowing of coronary
arteries, usually by atheromas ( ANGINA and MI)

(1) – Angina: - Classically centralized pain/tightness/heaviness of Some patients with atypical


the chest that radiates to the jaw/arm/neck pains, particularly diabetic
- Exacerbated with exercise, Relieved with rest

Stable angina – patient with chronic symtpoms (predictable relationship to physical excursion causes
symptom onset of transient pain <10min)

Patient should bring medication (glycerol trinitrate spray, to the dental appointment

(2) – Myocardial infarction: - Symptoms similar, but more extreme, than that of angina.
- No relief of the symptoms

**Someone not previously diagnosed, of the symptoms appear to be worse than normal:
Consider as MI
Dental considerations for patients with coronary artery syndrome
Major dental procedures – defer to at least 3 months after an attack (unless an emergency)
o Ensuring that their current condition is stable

All patients will be on aspirin and other antiplatelet agents


 Increased risk of bleeding

Avoid stopping aspirin or other preventative medications (due to risk of attack)

Clopidogrel – must be maintained 6 weeks post bare metal stent (if not a clot may form around the stent –
cant stop otherwise may induce stent thrombosis)

Local adrenaline use: no issues, however consider drug interations (consider dosage with beta blockers,
arrhythmias)

Patients with angina should bring their medication (glycerol trinitrate to the apt.)

--Pace maker devices – dental electronics doesn’t induce intereference with modern decivse (IM
implanted)

MEDICAL EMERGENCY: Angina attack/Myocardial Infarction

For someone who has been previously diagnosed, or pain more extreme:
- Cease treatment
- Administer glycerol trinitrate spray or tablet sublingually (vasodilator that relieves the
symptoms)
 Spray: 400μg sublingually up to 1200μg
 Tablet: 600μg sublingually up to 1600μg
 Every 5 minutes, 2 times, on the 3rd time call 000
- Refer the patient to their GP

For someone who has been never had a previous diagnosed, or more severe than normal: Myocardial
infarction:
- Stop dental treatment
- Call 000 …assume the worst…
- Administer oxygen
- Chew one aspirin tablet (300 mg) – provided patient hasn’t had medication already
- If falls unconscious and required, basic life support administration
Consider medications a patient may be taking:

Very common for a patient to be taking anticoagulant (most common warfarin) and antiplatelet (most
common aspirin, clopidogrel after stent, prasugrel) drugs

Issue with patient not clotting after treatment (bleeding risk) if the drug is not stopped
…if drug is stopped, risk of thromboembolic event (potentially more serious)

1) Antiplatelet agents

Aspirin (NSAID)– mild blood thinning action, antiplatelet agent reducing platelet function and
therefore blood clotting
 Patients tend to be more likely
Blood thinner when <300mg, pain relief when >600mg to bleed more
 Increased risk of bruising (warn
COX inhibitor – reducing prostaglandin synthesis pt)
 Local measures should be used
Acting time – 7-10 days – If ceased, at least 7 days in to overcome local bleeding
advance, commenced 2 days (suturing, cotton balls, gauze)
after

Clopidogrel or prasugrel Commonly used in association with aspirin to prevent thrombosis


at a stent, or used if a patient is unable to take aspirin
…6 weeks post bare metal stent, up to 1yr after some stents
-> Preventing myocardial infarction, stroke

 Do not cease clopidogrel/prasugrel –


consult specialist
 Use local hemostatic measures, warn
patients of bruising risk

2) Anticoagulants

Warfarin: Vitamin K antagonist (inhibits Vitamin K production in the liver – inhibiting production
of vitamin K dependant clotting factors II, VII, XI, X)
…platelet aggregation inhibitor

Used in: blood clots occurring in the lungs/atrial fibrillation


Most mild procedures can be carried out with the patient on warfarin

Check the patients INR (internationally normalized ratio – indication of how thin the patients blood cots,
prothrobin time – how long it takes the blood to clot) within 24 hours prior to treatment

Check INR within 24 hours of treatment


… < 2.2 no contraindications, proceed
… 2.2 - 4.0, precede using transexamic acid mouthwash protocol
… > 4.0 do not proceed; refer patient to medical practitioner

Transexamic acid mouthwash protocol: Used if the patients INR is between 2.2 and 4.0

During surgery –
- Obtain bottle of 4.8% tranexamic acid mouthrinse (if not available crush 500mg tablet and
rissolve in 10ml water)
- After extraction: Flush out extraction sites, irrigate sockets with transexaemic acid
- Fill socket with loosely packed haemostatic agent -> suture -> patient bites hard on gauzed filled
with transexaemic acid
After surgery – patient prescribed transexaemic acid to rinse with 10 ml for 2 minutes, 4x day, 2-5
days
…review appointment in 2 days

OFMS Medical EMERGENCY: PERSISTENT BLEEDING:


 Types May be related to medications, or
o Primary hemorrhage – occurs at time of systemic bleeding conditions (Hereditary
the injury/operation (surgical trauma) / Acquired / Uncontrolled hypertension)
o Late onset bleeding:
- Delayed: (24-48 hours post)
o (reactionary – slipping ligature, clot displacement)
- Secondary (5-10 days post)
o Due to an infection / blood clot breakdown
 Management:
 (good illumination, adequate retraction, thorough suctioning – determine source of bleed)
 Look for signs of excessive blood loss (pallor, lethargy, weakness, unconsciousness)
o Initial screen and management
 Don’t panic, manage pt. anxiety, reassure, sit patient upright
 Tell patient not to rinse and spit, don’t bite on paper towels (handkerchief)
o Local hemostatic agents - Control the bleeding site
  Packed, clamped, irrigation, debrided, sutures, pressure (v. important – bite
hard on gauze for 15 minutes)
…Possible use of haemostatic agents
o Possibly surgical management methods:
- Local haemostatic measures:
o Haemostatic agents – tranexamic acid
o Pressure swabs
- Vasoconstrictors in Las - Ligation of blood vessels (block blood vessels)

Possible hospital administration if uncontrolled, close vitals monitoring, treating co-morbidities, fluid management, hematology team consult)

If bleeding doesn’t resolve with 15 minutes of compression:


…calculate blood loss (saturated 10x10cm gauze=5ml blood), if over 50-100ml use local
haemostatic measures)

Assess site of bleeding (light, suction, retraction)


-> Infiltrate LA with vasoconstrictor at the area, pack socket with local haemostat + suture
Clexane: Subcutaneous injections accelerate antithrombin III to inhibit coagulation factor Xa
Active for 12-24 hours

NOACs – New oral anticoagulants: Thins the blood as much as warfarin

Stopping patient anticoagulation medication

All anticoagulation medication increases the patients risk of bleeding on treatment

We are able to take the patients off their anticoagulation medication, however we
need to assess the risk of the patient suffering a thromboembolic event

…Discuss with the patients specialist/GP

3) ACE inhibitors – Consider Triple whammy with diuretics and NSAIDs


4) Antianginals
5) Antiarrhythmics
6) Duiretics

(3) – Heart failure / Congestive heart failure / congestive cardiac failure


The end stage of a number of conditions (not all cardiac)
(ischemic heart disease, hypertension, diabetes……)

A maladaptive response with the body trying to maintain


blood pressure
…body reponsed increasing soium and water
retention

…Resulting from any structural or functional cardiac disorder that impairs the ability of the
ventricle to fill or eject blood
…Disease of the elderly

 Predominantly of the left vertical with pulmonary congestion and dysphonia


 Or predominantly of the right ventricle with elevated venous pressure, peripheral oaedema,

Clinical implications of heart failure:


Clinical manifestations of heart failure:
Treatment only carried out if the patients heart -> Asymptomatic
failure is stable -> Congestive state – oedema in the systemic
or pulmonary system (oedema in the lungs
Often cant tolerate a horizontal position, should can occur very quickly)
be placed with the head higher than the heart -> Low output state – exercise intolerance,
…A good gauge is how many lethargy, weakness etc
pillows they sleep with -> Cardiogenic shock – heart pumping is so
impaired that delivery to the body and the
NSAIDs should be avoided as can worsen heart brain is not adequate
failure …ultimately death
Valvular heart disease Damage or defect of one of the heart valves
…Aortic valvular heart disease
…Mitral valvular heart disease
Valvular heart disease may be:
…Stenotic – resulting in the narrowing of valves
…Regurgitant – leaking heart valves, fail to close correctly

Aetiologies of valvular heart disease:


 Degerative
 Calcification
 Rheumatic fever – particularly involved in mitral stenosis
 Infective disease resultant of a streptococcus infection
 Can involve the heart, joints, skin and brain
 Congenital
 INFECTIVE ENDOCARDITIS

Clinical manifestations of valvular heart disease


 Asymptomatic
 Cardiac mumurs Dental considerations:
 Heart failure (must be severe to result in the following): - Medications
 Fluid retention, dysphonia, exercise intolerance - Increase infective
 Arrhythmias endocarditis risk
 Infective endocarditis

Arrhythmias
Atrial fibrillation / Supraventricular tachycardia:
 Dentistry complication: anticoagulation (tendency to bleed)
 Atria are not filling correctly before they beat (increased HR)
o Small clots can form and travel to the ventricles and into circulation (possibly causing a
stroke)
o Common use of warfarin as a blood thinner
Pacemaker: Implanted electrical devise that produces
Ventricular arrhythmias:
electrical impulses that stimulate the heart.
 Tachyarrhythmia – if prolonged can
lead to cardiac arrest
Certain dental devices can introduce electromagnetic
interference (EMI) by causing single beat inhibition
Conduction defects –may require a
on pacemakers programmed to unipolar sensing mode.
pacemaker
…Devices found to have an effect:
Question 5: Do you have a pacemaker? -- Magnetostrictive ultrasonic scalers
(e.g. Cavitron) / Electrosurgery unit
…Devices found not to have effect:
Dental considerations of pacemakers and --Piezoelectric scalers (EMS)/EAL /EPT
other implantable cardiac devices Precautions should be taken for pt with pacemakers:
 Keep working end and cabling > 6
Do not present issues during dental treatment. inches from the implanted device
 Do not drape cords over pt chest
If devices are intramuscular (most modern o If inhibition occurs:
appliances), then dental electronics do not  Symptoms similar to those
interfere experienced prior to device
implantation (dizziness, light
Endocarditis is not at increased risk because of headedness etc.)
intramuscular placement  Switching off unit will stop the
interference  device will
Ultrasonic scalars are an issue if the pacemaker is automatically re-pace as usual
not intramuscular (not modern appliance)
Hypertension High blood pressure (systemic >140)(diastolic >85)

Primary (resultant with absence of other disease)


…Vs…
Secondary (resultant a result of the presence of other systemic disease)

A major risk factor to ischemic heart disease, congested cardiac failure, stroke
…a higher pressure  stretch of artery walls  risk of rupture

Dental considerations for patients with hypertension:

Stable and controlled is not usually an issue

- Measure BP before apt.: >180.100 mmHg (malignant hypertension), DEFER treatment

Dental procedures may cause a rise in blood pressure possibly to a dangerous level
 Provision of analgesics post treatment , possible sedation with severe
dental phobias

Consider the influence of NSAIDs (e.g. ibroprofen and aspirin)


…may cause salt and water retention due to their effect on the kidneys
--> This may increase the blood pressure
**Use the lowest possible for shortest time to reduce the effect**
- LA use probably has very little effect on blood pressure
- Prompt treatment of severe longstanding dental pain

Congenital heart disease

Cyanotic – results in the deoxygenating of blood - patient in a state of cyanosis


……………vs…………… Acyonotic

Dental considerations for patients with congenital heart disease:

Consider antibiotic prophylaxis due to increased risk of developing bacterial endocarditis

Patients with complex congenital heart disease:


…Undertake dental procedures in hospital to have available support networks in
the case of an emergency

Vasovagal syncope
 The most common cause of syncope due to the loss of vagal imput into the heart causing
hypotension
 Commonly trigger involved (something pt is scared of – needles)

Prodromal symptoms of vasovagal syncope: typical symptoms just prior to fainting.


 Feeling faint
 Sweating
 Looking pale
 Nauseous
 Anxious
MEDICAL EMERGENCY: Vasovagal syncope

Stimulus: e.g. needles


…avoid showing the patient the needle, keep them calm and distract them with music,
talking, shutting their eyes

On prodromal symptoms – if patient feels faint:


- Cease dental treatment
- Lie the patient flat to avoid possible trauma
- Raise patients legs
- Assess consciousness

On fainting unconscious:
 Lie the patient horizontal (measure blood pressure and pulse rate)
 Place on their side, elevate their legs (patient should regain consciousness rapidly)
 Allow the patient time to recover
 Drink plenty of water
 Cool the patient with a damp cloth

 If it is the patients first time fainting and they are older, consider referral to GP

FAILURE TO WAKE _ CALL 000 and COMMENCE BASIC LIFE SUPPORT

Cardiac arrest: Sudden loss of consciousness, heart Dental emergency - cardiac arrest :
stops beating, stops breathing - Cease dental treatment
- Call 000
- Initial basic life support
 Generally due to arrhythmias (verntricular
tacchicardia or ventricular fibrillation

Heart Murmur Audible disturbances of blood flow - associated with valves functioning abnormally.

Aetiology
- Pathological abnormalities/conditions
Dental considerations of heart murmur:
- Valvular disease
**INDICATES for risk of underlying heart condition
- Mitral valve prolapses
- Warns for risk of colonization of damaged
 Backflow of blood from the
valves by blood borne bacteria
ventricle into the atrium
o Indicates possible AB prophylaxis
- Rheumatic heart disease
requirement
- Previous IE
...If high risk condition e.g. complex
- Prosthetic heart valve
cyanotic heart defect, prosthetic
- Congenital cyanotic heart disease
heart valve
Rheumatic fever: Inflammatory diseae that follows an upper respiratory tract Streptococcal
infection
Can involve the heart, joints, skin and brain
Dental considerations of rheumatic fever:
- If patient is also considered high risk (e.g. indigenous)
o Require Antibiotic prophylaxis
Question 6: Have you ever had heart valve, or open-heart surgery?

 Determine information about this:


 What was the surgery?
 When was this completed?
 ……Defer elective treatment 3 months after surgery
 Was a prosthetic material used? ==> AB prophylaxis requirement

Dental implication:
- Stent placement  pt will likely be on clopidogrel / prasugrel to prevent stent thrombosis
o ….Do not cease mediation
- Defer elective treatment 3 months after placement
- AB prophylaxis requirement (prosthetic material used in repair)

Question 6: Have you ever had


INFECTIVE ENDOCARDITIS AND AB PROPHYLAXIS:
heart valve, or open-heart surgery?
Infective endocarditis: Inflammation/infection of the inner layers of the heart (endocardial
surface), typically the heart valves

 Uncommon – but high morbitity and mortality (life threatening)


 Possible infection with Streptocuccus and Staphtococcus
 Treatment very difficult (possibly requiring heart valve surgery)
 Few cases clinically requiring AB prophylaxis (specific patients and high risk proceedures)
 [more] likely to arise from daily activities (importance of good OH to those at risk)

Symptoms include: (incubation period ±2weeks)


 Unexplained fever
 Night sweats
 Malaise / lethargy
 Generally unwell
 Other
o Janeways lesions
o Splinter haemorraging
Question 16: Do you have any Joint problems, Arthritis, or history of joint replacement surgery?

- PROSTHETIC JOINTS: Used only if there is dental problems within the first 3 months
since placement
o But if perious experience should be provided
**Elective treatment should be deferred 3-6 months
ANTIBIOTIC PROPHYLAXIS: Prophylactic administration for prevention of infection in situations
where there is significant risk of infection occurring

NOW RECOMMENDED for only for patients with cardiac conditions with high risk, undergoing specific
high risk procedures…
… … … HIGH RISK PATIENTS … … …

 Prosthetic cardiac valve, or prosthetics material used in repair


 Previous experience of infective endocarditis
 Congenital heart disease, ONLY if:
o Have unrepaired cyanotic defects
o Completely repaired with prosthetic materials
 Within 6 months of placement
 After this valve has been endothelialised
o Repaired defects with residual defects
 Cardiac transplant with subsequent development of cardiac
valvulopathy
 Rheumatic heart disease in HIGH-RISK PATIENTS, including…
o Immunocompromised patients + indigenous Australians
 Immunosuppression with medications
(corticosteroids, chemotherapy)
- No longer recommended for patients with other forms of valvular/structural heart
disease (including mitral valve)

… … … UNDERGOING HIGH-RISK PROCEEDURES … … …

High risk procedures: Moderate risk proceedures:


- Extraction Consider if multiple procedures being
- Periodontal procedures: completed, or periodontal disease is present
o Subginginval scaling / root - Full periodntal charting
planning - Intraligamentary / intraosseous LA
o Surgery - Supraginginval calculus removal
- Replanting avulsed teeth - Rubber dam pleacement with clamps
- Oral surgery: o And interproximal wedges
o Implants - Restorative matrix band/strip placement
o Apicectomy - Endodontics beyond the apical formamen
- Placement of orthodontic bands
- Placement of supraginginval retraction
cords

Low risk: Not required


| Oral examination | Infil/block injection | Restorative dentistry | Supragingival rubber dam
clamp | Intracanal endodontic procedures | Suture removal | Impression taking | Radiographs |
Antibiotic prophylaxis - The critical period for antibiotics to be in the system is at the time
of infection, and the 4 hours following entry

…Standard administration…
Amoxycillin: Dosage: 2g (child: 50mg/kg up to 2g), orally 1 hour prior, IM 30 minutes prior, IV
immediately prior
⏎Amoxycillin(Oral) / ampicillin(IV) (Aminopenicillins)
 Moderate spectrum
 Active against gram positive + some gram negative
 Contraindications:
 Inactivated by beta-lactimase
 ***Can be assisted with clauvanic acid – makes resistant***
 Addition of ^^clauvanic acid^^ broadens the spectrum
 Indications:
 ANUG (500mg, every 8 hours, 5 days) – only in severe cases, immunocompromised pt., or
unresponsive to tx
 Antibiotic prophylaxis – (2000mg orally 1 hour prior, IM 30 min prior, IV immediately
prior)
 Spreading infections, periodontal abscesses, Dentoalveolar surgery infections, tooth
avulsion
…Patients with immediate hypersensitivity to penicillin…
Clindamycin (orally or IV)
Dosage: 600mg (15mg/kg up to 600mg), orally 1 hour prior, IV over the 20 minutes prior

Lincomycin (people who are unable to swallow) (IV)


Dosage: 600mg (15mg/kg up to 600mg), IV over the 1 hour prior
⏎ Lincosamides (clindamycin, lincomycin)
 Active against gram positive and anaerobic cover
 Reserved for:
o MRSA
o Patients with a penicillin allergy
 Clindamycin:
o Indications:
 Antibiotic prophylaxis (600mg orally, 1 hour prior)
 ANUG (immunocompromised/severe cases/unresponsive to tx AND penicillin allergy) (300mg orally, 8
hourly, 5 days)
 Surgical infections
 Spreading odontogenic infections / Salivary gland infections (450mg IV, 8 hourly)
 Periodontal abscess (300mg orally, 8 hourly, 5 days)
 Lincomycin:
o Indications:
 Antibiotic prophylaxis (600mg orally, 1 hour prior)
 Spreading odontogenic infections / Salivary gland infections (450mg IV, 8 hourly)

Vancomycin Dosage: 25mg/kg up to 1.5g (child <12 30mg/kg up to 1.5g), slow IV infusion
ending just prior
⏎ Vancomycin (GLYCOPEPTIDES)
 Active against gram positive only
 Uses:
o Antibiotic prophylaxis (patient with penicillin hypersensitivity) (25mg/kg up to 1500mg IV immediately prior)
o Only used for treatment of MRSA
o Treatment of penicillin resistant organisms
 Doesn’t target in the way penicillin does
 Low bioavailability (doesn’t get into systemic circulation through oral administration)
o  Administered IV (unless targeting C difficile in the GI tract)

Teicoplanin Dosage: 400mg (10mg/kg up to 400mg), IM 30 minutes prior, IV immediately


prior

…patients with hypersensitivity (not immediate) to penicillin… (as cephalexin is synthetic penicillin)

Cephalexin Dosage: 2g (child: 50mg/kg up to 2g), orally 1 hour prior

⏎ Cephalexin / Cephalothin Cephazolin (1st generation)


 Moderate spectrum
 Active against gram positive + some gram negative
 Streptococcus, staphylococcus
 **Alternate for penicillin hypersensitive pt** - Generally well tolerated
 Generally only minor reactions though – still a 1/10 chance that they are also allergic to cephalexin
 Uses:
o Cephalexin: Antibiotic prophylaxis if penicillin allergy (2000mg orally, 1 hour prior)
o Cephazolin: Deep odontogenic infection into submandibular and pharyngeal spaces (2000mg IV, 8 hourly)

Bacteremia:
…Most dental treatment involving the blood
 Transient (cleared in 30 minutes ) bacteremia
…Similarly, may be induces by at home OH
HAEMATOLOGICAL DISORDERS:

Quantitative abnormalities

 Anaemia – Decreased level of circulating hemoglobin causing impaired oxygen


carrying capacity (reduced RBC or problem with red blood cells)

Required for RBC production:


- Bone marrow activity (low, not replacing), haematnitics (inadequate building blocks /
minerals for RBC production), erythroprotein (hormone inadequate to stimulate production),
general wellness (anabolic state for production)

…Required normal genes: > 4 components to the haemoglobin structure


…Each has 1 iron bonded for the carrying of oxygen

Symptoms: -> lethargy (unreliable = heart failure, angina, pallor, brittle nails)
Mucosal pallor, oral ulcerations, glossitis, andular cheilitis)

Types: - 5 major types


 (Normochronatic, normocytic) – anaemia of chronic disease
 (Microcytic – incl. THALLASAEMIA) – iron deficiency anaemia
 (Macrocytic) – low vitamin b12, folate deficiency
 (Polychromatic)
 (Very abnormal cells)

Clinical implications of anaemia on dentistry

1) Reduced cellular activity due to poor oxygen supply


 Poor wound healing (effecting treatment)
…Prior to treatment may administer antiobiotic prophylaxis for more
major treatment
 Poor dental hygiene and poor response to dental treatment

2) Mucosal Pallor - The redness of the mouth is not as red as it should be if the patient has aneamia
- Dentist may be first to recognose anaemia -> referral

3) Mucosal pallor, oral ulcerations, glossitis, andular cheilitis


…Opportunistic infections more prevalent – e.g. oral candidiasis

 Thalassemia- Mutation in Alpha or Beta chain of haemaglobin  abnormal hemoglobin


Leading to anaemia
Genetically inherited: Autosomal recessive

Symptoms: Anaemia, heart problems,


Cardiomyopathy (risk of collapse with GA)
Bone marrow expansion  maxillary enlargement
 Polycythaemia – Overactive bone marrow (primary) or excessive EPO production
(secondary) resulting in too much haemoglobin (problem with RBC)

Clinical implications of polycythaemia on dentistry


Capillaries become overfilled with blood
 Resulting in excessive bleeding

 Neutropenia – reduced neutrophil count


(neutrophils normally involved in the phagocytosis of bacteria)

Causes: congenital or idiopathic, OR acquired (marrow suppression, neutrophil destruction, liver


failure)
Clinical implications of neutropenia on dentistry
May be unable to fight infection
…Invasive dental procedures risk of post-op infection / IE

 Thrombocytopenia – low blood platelet count


(assisting in the blood clotting mechanism)

Symptoms: Purpural (large bleeding areas) and petechiae (small pinpoint


bleeding – can be found on palate)

 Haemophilia – deficiency in clotting factors

Haemophilia B (Christmas disease) = factor IX deficiency


Haemophilia A = factor VIII deficiency
…Levels of 25% of normal will likely have adequate clotting
… Levels <5% with have symptoms of abnormal bleeding

Treatment:  Increase factor VIII production => with DDAVP


…Supplementation can be completed with replacement factors (genetically modified
recombinant factors)
…Antifibrinolytic therapy is useful in the post surgical phase to protect the formed clot
-- bleeding risk

 Leukaemia (Lymphomas and myelomas) - Neoplastic proliferation of white blood cells


Arising from bone marrow
Classifications:
1. Acute vs. Chronic 2. Myeloid vs. Lymphoid

Clinical implications leukaemia on dentistry


Complications of leukaemias in arise from:
1) Cytopenias including: anaemia – neutropenia – thrombocytopenia
2) Immune deficiency – DENTAL TX completed between chemo sessions

Oral manifestations: ANAEMIA, BLEEDING AND INFECTION ARE


OVERWHELMING RISKS
- Decreases resistance to infection
o Increased risk of candidiasis or herpetic infection
- Increased bleeding
- Petechial (pinpoint) haemorrhaging (even with minimal trauma)
- Gingival swelling
- Ulceration
- Mucosal pallor
Diseases in coagulation

 Von Willebrans disease – Combined (von willebrans) factor VIII and platelet disorder

….Normally factor VIII forms cross-links, tightening the clot


Symptoms:
 Asymptomatic
 Mucosal purpura
 Treatment includes
o DDAVP – promote the release of von willebrans factor and subsequent increase
in factor 8
o Transexamic acid – antifibrinotytic
o Factor 8 which helps in the clotting process
RESPITATORY CONDITIONS

OBSTRUCTIVE LUNG DISEASES:  asthma, COPD

Question 8: Have you ever had tuberculosis, asthma, or any other lung diseases?

Asthma – Chronic inflammatory disorder of the airways associated with airway hyper-
responsiveness
…leads to recurrent episodes of wheezing, breathlessness, chest tightness, and coughing

An obstructive lung disease --- constriction of the airways …due to muscle spasms
and airway oedema
 Very common: 10% of adults, 11% of children)

Hallmarks of asthma = (1) Bronchoconstriction (2) Mucous hypersecretion (3) Airway oedema
2 types of asthma: Questions to ask:
 Intrinsic (non-allergen)  How long have you had asthma for?
o Medication induced  What triggers your asthma?
o Anxiety
 Are you taking any medication?
o Stress, Smoke, viruses
 Do you use a spacer?
o Exercise
 Did you bring your reliever today?
o Cold air/ dry air
 Are you anxious about treatment today?
 Extrinsic (allergic)
o Inhaled allergen –dust mite
allergen, pet dander, pollen
Treatment of asthma:
Relievers - Act to relac smoot muscle around the bronchioles, providing instant relief (short
acting beta-2 agoist)
 Ventalin inhalers (salbutamol - Blue, terbutaline)
- short acting beta-2 agonists -> smooth muscle relaxation

Preventers - taken to prevent release of inflammatory mediators causing constricting airways


> If patients have more frequent attacks – corticosteroids reduce airway
swelling and hyper responsiveness (fluticasone)
 Flixotide (Red cap, orange body)(^ fluticasone^ only)
 Seretide (Purple)(^ fluticasone^ + Long acting Beta-2 agonist)
(e.g. salmeterol)
 Salmeterol (Green)(long acting beta 2 agonists – salmeterol)(Serevent)
 Prednisalone (>3 week use may induce adrenal suppression – pt. may
not be able to cope with stress  Adissonian crisis - **increase
corticosteroid dosage)
Dental implications: SAMTERS TRIAD -
 Avoidance of triggering an attack (anxiety may induce)  Aspirin sensitivity
…Patients told to bring inhalers with them  Nasal polyps
 Use of rubber dam? – dependent on severity due to airway obstruction  Asthma
 Several drugs (NSAIDs) can cause bronchoconstriction **ASPIRIN SENSITIVITY IN
 Can exacerbate issues with asthma 10% OF ASTHMATICS**
…Paracetamol is the analgesic of choice (adverse reactions are more rare)
 Patients at risk of developing oral candidiasis due to inhaled
corticosteroids
…pt advised to rinse with water and spit after use
…or the use of a spacer is advised
 Beta-2 agonists – decrease saliva flow, plaque pH -> gingivitis/caries,
increases symptoms of GORD -> erosion
MEDICAL EMERGENCY: ASTHMA ATTACK

Pt advised to bring their bronchodilator medication to the appointment (often plan in place in event of
attack)

Management varies depending on the severity of the attack:


--- Initial assessment of the attack immedicately

Mild attack: Indicated with pt able to talk in scenetences, pulse <100bpm


1) 4 puffs of short acting bronchodilator:
…Via spacer: 1 puff at a time, pt takes 4 breaths in and out of spacer after each puff
…No spacer: Pt holds breath for 4 seconds after each puff
2) Wait 4minutes (no improvement repeat (1) administering 4 additional puffs)
NO IMPROVEMENT  manage as moderate/severe attack
PATIENT RECOVERS  temporize dental state, make anoth appt. for completion, when pt fully
recovered discharge

Moderate attacks: Indicated with pt talking in phrases, HR: 100-120 bpm (children 100-200), hospital
admission is probably required
Severe attacks: Indicated with pt only able to convey words, HR >120 (children >200bpm), physical
exhaustion and altered consciousness, accessory muscle use, hospital admission required
1) Call 000
2) Oxygen provision at 6L/min
3) 4 puffs of short acting bronchodilator:
…Via spacer: 1 puff at a time, pt takes 4 breaths in and out of spacer after each puff
…No spacer: Pt holds breath for 4 seconds after each puff
4) Wait 4 minutes. Continued to give 4 puffs every 4 minutes

 Relationships
o Asthma and Dental caries:
 Decrease salivary flow caused by beta 2 agonists and increase in Lactobacilli and
S. mutans
 Fermentable carbohydrates present in anti-asthma medication
 Increase in frequency of consumption of cariogenic drinks due to excessive
thirst
 Some dry powder inhalers contain surfer so the pt. can tolerate the taste of the
drug
o Asthma and Dental erosion:
 Reduction in buffering capacity and salivary flow rate due to beta 2 agonist
 Increase in exposure to teeth to acids (acidity of medication, soft drinks and
GORD)
 Certain inhaled beta - 2 - agonist drugs can decrease lower oesophageal
sphincter pressure - associated with GORD
o Asthma and Periodontal disease:
 dehydration of alveolar mucosa due to mouth breathing - increase in
consumption of drinks to compensate oral dehydration.
 alteration of immune response and increase concentration of IgE in gingival
tissue
 Decrease in bone mineral density associated with inhaled corticosteroids
o Asthma and Oral candida:
 generalised immunosuppressive and anti-inflammatory effects of steroids
 higher salivary glucose conc. could promote growth and proliferation of Candida
Chronic obstructive pulmonary disease – Chronic airflow obstruction that is not fully reversible

Emphysema  Destruction of the alveoli , enlargement of air spaces(lung parenchyma is structurally


Pink puffer damaged)

Chronic  Chronic excessive mucous production and persistent productive cough


bronchitis  Sputum production and chronic cough on most days for more than 3 months of
Blue bloater the year

 Airflow obstruction is often progressive, and associated with abnormal inflammatory


response
Alpha-1-antitrypsin deficiency: -> Reduced action of neutrophil elastase
causing reduced elasticity
…Inherited and often associated with smoking

Normally Alpha-1-antitripsin prevents trypsin from breaking down


elastic fibers

Aetiology: Tobacco smoking | Long-term exposure of occupational pollutants

Symptoms:
- Chronic cough with sputum production
- Persistent dysphonia worsening with exercise
- Chest discomfort
- Co-morbidities  CVD, respiratory infection, osteoporosis, fractures

Treatment of acute COPD:


 Oxygen titration and portion – to maintain blood O2 levels
 Bronchodilators
 Steroids – corticosteroids used systemically
 Non-invasive ventilation – maintains the airways open to ‘flush’ lungs

**With emphysema we are unable to regenerate lung tissue – so treatment aims to maximally use
remaining tissue and stop further destruction**
Treatment of chronic COPD:
 Cessation of smoking
 Spiriva – long acting muscularinic antagnoist results in dilation of the airways
 Pulmonary rehabilitation - chemotherapy based
 Physical therapy and maintaining fitness – (improve exercise tolerance)
 Lung transplantation

Dental implications:

 Patients do not tolerate horizontal position

 …Patients taking corticosteroids should increase dosage if they have adrenal


suppression

 All health professionals should advocate smoking cessation (5 A’s, 5 R’s).

 Patients at risk of developing oral candidiasis due to inhailed corticosteroids


 …patient should rinse with water and spit after use
RESTRICTIVE LUNG DISEASE ---->

(1) – (Idiopathic) pulmonary fibrosis:


^Most common form^

Symptoms: - Breathlessness, Dry cough --> Fatal

(2) – Interstitial lung disease:


-> Sarcoidosis - inflammatory causing granuloma inflammation
-> Pneumoniaconoisis – commonly associated with occupational dust exposure
-> Hypersensitivity pneumonitis – reversible immune based reaction
-> Connective tissue disease – systemic autoimmune disease causing destruction of connective
tissue

(3) – Chest wall disease: Deformities of the cage of the lungs may cause restriction limiting
its compliance, and effect mechanics of breathing
-> Kyphosis
-> Pectorus excavatum

(4) – Pleural disease:


 Pleural effusion (pleura inflamed reducing lung volume, fluid buildup between
chest and lungs)
 Mesothelioma (cancer growing around lining)
 Neuromuscular disease: Weaken intercostal muscles/diaphragm

(5) – Pulmonary vascular disease


Acute: Pulmonary embolism - Blood clots form in pulmonary arteries
 Breathlessness, chest pain, dizziness, hypertension, choughing blood
… Sudden onset of hours to days (often after sedentary behavior e.g.
treatment – blood clot travels through blood stream  lungs))

Chronic pulmonary hypertension: Remodeling and thickening of the walls of the lungs
-->Elevation of pulmonary blood pressure
--> Right sided heart failure

Lung cancer
- Poor prognosis – usually late detection (more common in smokers)
- Associated with hemoptysis - coughing up of blood

Small cell cancers: Chemotherapy is used as a first line, poor prognosis


Non-small cell cancers: -Majority
--> Adenocarcinomas and squamous cell carcinomas
-Poorly differentiate large cell carcinomas
Respiratory tract infection

Upper airway:
 Pharyngitis
 Tonsillitis
 Epiglottitis
 Conguncto-rhino-sinusitis

Lower airway:
 Laryngo-trancheo-bronchitis

Lungs:
 Pneumonia (opportunistic bacterial infection)

Obstructive sleep apnea Characterized by repetitive obstruction to pharyngeal activity, leading to


episodes of apnea (cessation of breathing) during sleep

Cause: Collapse of the pharynx during sleep, with loss of breathing for a time greater than 10s
…Due to an imbalance between pharyngeal dilator muscles
…Also some degree of oedema from repositioning of fluid whilst lying down
--- Reduction in oxygen causes a microarousal of sleep (so that tone is restored)

Risk factors: -> Obesity, Male predominance, age, Alcohol and smoking
-> Retrognathic mandible or retrusive maxilla Reduction in the diameter of the
pharyngeal space

Symptoms:  tiredness,  CV disease (MI, hypertension, stoke), accidental death

Treatment
= SLEEP HYGEINE: Regulate and produce effective sleep (exercise, routines, meals, napping,
weight loss, drugs and alcohol)

= CPAP (Continued positive airway pressure) – pressure holds the airways open (po
- Poor compliance, 100% effective when worn

= MANDIBULAR ADVANCEMENT SPLINTS - protrudes the mandible preventing collapse


- Better compliance, not as effective
o  Some orthodontic effects, dry mouth, gum irritation, TMJ discomfort
Dental implications:
- Dentists have role in producing mandibular advancement splints (working with respiratory team)
o Use of oral devices without diagnosis is NOT appropriate

- Increased risk of respiratory arrest under general anesthetic


Hyperventillation syndrome Occurring when a patient hyperventilates, commonly associated
with anxiety or acute panic attacks

Symptoms: (Can be confused with syncope, early phases of acute asthma attack or MI)
 Rapid breathing, occational deep breaths
 Rapid pulse
 Altered consciousness
 Lightheadded, dizziness, blurred visionn
 Shortness of breath
 Feeling of panic
 Tingling in the toes/fingers
 Carpopedal spasm – contraction of wrists, hands, ankles

MEDICAL EMERGENCY: HYPERVENTILLATION SYNDROME


- Cease dental treatment
- Encourage patient to slow their breathing
o Firmly reassure and calm them
- Rebreathe expired air: Have patient cup their hands over nose and mouth
o **DO NOT GIVE OXYGEN <<prolongs symptoms

- Not resolving:
o Reconsider diagnosis (Asthma, analphylaxis, MI)
o Call 000 is Persisting more than 5-10 minutes, or carpopedal spasm is
extensive

Inhailed / swallowed objects:

Passage of inhaled objects:


- Foreign object in the lungs must be urgently removed
- Passing through the GI tract may be uneventful

Prevention: Rubber dam application, do not rush a procedure, tie dental floss around objects prone
to being swallowed, use high volume suction
…IF an object is dropped – IMMEIATELY rotate head to the side (object can fall out)

Obstruction of the airway:


- Differentiate between partial and complete airway obstruction:
- Partial:
o Wheezing, noisy inspiration, labored breathing
o Coughing spasms
o Cyanosis (severe lack of oxygen
- Complete:
o Inability to break/speak/cough
o Agitation, gripping of the throat
o Cyanosis
o Bulging of neck veins
o Rapid development of respiratory failure (then cardiac failure)
o Loss of consciousness
MEDICAL EMERGENCY: INHAILED OR SWALLOWED OBJECT
In the event an object has appeared to fall down the oropharynx…
- Cease dental treatment
- Check whether the object in in the mouth/fallen elsewhere, + remove
- NOT FOUND – move pt. upright
- Check vital signs
- **DO NOT allow to drink
- Chest X-ray within 1 hour
o Object within the lungs should be removed by bronchoscopy/thoracotomy
- If patient has swallowed sharp objects (needle, bur, files) and develop abdomen pain
o Urgent medical referral - Abdomen radiograph

PARTIAL AIRWAY OBSTRUCTION


 Call 000
 Reassure pt, encourage them to relax, breathe deeply
 Try to dislodge by coughing
 Pt unable to cough
o Up to 5 back blows between the shoulder blades using the heel of the hand
 Check effectiveness of each blow between

COMPLETE AIRWAY OBSTRUCTION


 Call 000
 Turn pt. to the side
 Attempt to clear/open the airway by manually removing
Check for signs of breathing – no signs:
 Give up to 5 back blows between the shoulder blades using the heel of the hand
 Check effectiveness of each blow between
Check for signs of breathing – no signs:
 Give up to 5 chest thrusts (check effectiveness between each thrust – identical to CPR
but harder and faster)
 Check effectiveness of each blow between
If complete obstruction continues:
 Cricothyroidectomy indicated
o Extend head back to stretch neck  Palpate cricothyroid ligament  Incise
through the skin and ligament  Maintain airway until assistance arrives
SMOKING STATUS Question 9: Do you smoke?
 Questions to ask:
-- How long smoking? -- How many a day? --What / how do you smoke?
--When do you have your first cigarette of the day?

 Systemic effects   risk of:


 Lung disease (COPD, pneumonia, lung cancer)
 Cancer
 Premature skin ageing
 Stroke, MI
 Sudden death

 SMOKING CESSATION
The 5 A’s for tobacco cessation: Patients not showing readiness to quit, apply the 5 R’s:
Ask – at each appointment about smoking Relevance – why is it important for them
Assess – patients desire/readiness to quit Risk – undesirable outcomes, their perio disease
Advise – on dangers of failing to quit Rewards – personal and family rewards, and
Assist – efforts to quit, guide, encourage financial rewards
Arrange – follow up and support Road-blocks – discuss/recognize difficulties and
management of barriers
Repetition – complete at subsequent visits
 DENTAL SIGNIFICANCE

Condition Description
Squamous Cell  Clinically
Carcinoma o EARLY  leukoplakia, erythroplakia OR mixed (red + white), painless
non-indurated ulcers
o LATER  indurated painful ulcers
o Commonly on lip, tongue, FOM, salivary fauces, retromolar area
 Dx
o Radiographically -- Radiolucency if bone involvement
o Histologically
 Nuclear hyperchromatism, mitosis of prickle cell later, abnormal
mitosis, deep cell keratinization, loss of definition between basal
and prickle cells, diminished intercellular appearance, drop-shaped
rete pegs
 Tx
o Chemotherapy, radiotherapy
o Refer
Leukoplakia Predominantly white lesion that cannot be wiped off/attributed to any disease. There are
different types of leukoplakia, categorized according to their appearance variations:
 Homogenous – uniform, not raised
 Nodular – slightly raised with erythematous base
 Verrucous
 Proliferative verrucous
Rare/poorly defined multiple white lesions w/ VERY HIGH malignant risk
 Clinically
o Common elderly females
o Initially develop flat leukoplakias that over decades, progress to
verrucous or SC carcinoma; unable to remove surgically & recur
Speckled
 Clinically
o White flecks/nodules on atrophic erythematous base
Erythroplakia Predominantly red lesion or plaque with well-defined borders, the texture is soft and velvety
 Clinically
o Commonly on lip, tongue, FOM, salivary fauces, retromolar area
 Prognosis
o Carcinoma is found in ~ 40% of these lesions
Nicotinic White hyperkeratotic thickening of palate with red-centered minor mucous gland umbilicated
Stomatitis swellings within
 Clinically
o Red scattered inflammatory dots on the palate
o White hyperkeratotic plaques
 Tx
o Smoking cessation  resolves within weeks
Smoker’s Non-cancerous brown pigmentation often in the gingiva due to toxic substances initiating production
Melanosis of melanin
 Clinically
o Black/brown pigmentation of oral tissue, especially the lower gingiva
 Dx
o Biopsy if doesn’t heal post-smoking
 Tx
o Smoking cessation  resolves within 3 months to 3 years post-quitting
Hairy Tongue Elongation of filiform papillae forming thick hair-like fur along dorsal surface
 Clinically
o 1/2 cm long, brown-black coloured extensions on dorsum of tongue
 Tx
o Scrape hyperplastic papillae; cleanse dorsum w/ toothbrush
Halitosis Oral malodour (caused by suphur produced by bacteria)
 Dx
o Subjective measurement
o Smell air from pt. mouth and compare to pt. nose
 Tx
o Treat causative factor
Delayed/impai  Nicotine has vasoconstrictive abilities, leading to decreased blood flow and therefore
red wound influences wound healing
healing
Implant failure  Poor healing post-implant surgery
 Require sound periodontium which is normally compromised in a pt. smoking
Alveolar Clot disintegration exposing the alveolar bone lining (lamina dura) to the oral environment
osteitis (dry  Ensure post-op instructions are given to avoid re-occurrence
socket)  Signs/symptoms
o Exposed bone in socket
o Strong dull throbbing ache
o Halitosis
 Management
o Pain relief until normal healing
o Socket dressing alvogyl for pain relief but delays healing
o Saline to remove bacteria/halitosis
Caries  Smoking influences the amount and contents of saliva
 Shown to have a significantly higher DMF index vs. non-smokers
Periodontal  Pathogenesis:
disease (PDD) o Nicotine: causes  gingival blood flow (aid in proliferation of anaerobic bacteria) and
 inflammatory cytokines/PMNs (enhancing tissue breakdown)
 Affects the tx outcome after SRP and regenerative periodontal therapy
o Less favourable healing
 Less improvement when considering
o Pocket depth reductions & CAL
o Resolution of gingivitis
 Lang & Tonetti
o 1-19 cigarettes daily = MODERATE RISK
ENDOCRINE CONDITIONS Question 14: Do you have diabetes?

Diabetes - Unregulated levels of blood glucose resulting in states of hyperglycaemia + hypoglycemia


…due to the deficiency in secretion of insulin
5% of population are diagnosed
(1-2 million Aust.,
10% undiagnosed
Long term complications:
 Microvascular
 Neuropathy – vessel ischaemia, segmental demyelination,
 resulting in decreased perception of pain,
 decreased cardiac output,
 impaired motility of the GI tract
 Nephropathy – changes to the glomerulus effecting filtration
 decreased urinary function and syncope,
 Retinopathy – damage to vessels leading to the eyes
 Macrovascular
 Coronary heart disease (causes 80% of diabetes related deaths)
 Stroke – risk increases 4 times
 Peripheral vascular disease
 Damage to the brain, kidneys, nervous system, heart, blood vessels
Signs/symptoms: Polydipsia (thirst), Polyuria (urine), Polyphagia (excessive appetite), Weight
loss, Loss of strength, Irritability, Drowsiness, Malaise, Blurred vision

Classifications:
 Type I diabetes:  Pancreatic beta cells not able to produce insulin correctly.

…Auto immune response destroys the pancreatic beta cells (insulin not
produced at all or produced incorrectly)

Aetiology: Genetics, environmental (possible viral trigger)

Stages of onset:
Polyuria -> Polydipsia -> Weight loss -> Ketosis -> Ketoacidosis
^^High level of ketones in the blood as the body is producing
ketones^^
^^High level of ketones makes the blood acidic^^

Treatment: requires insulin injections

 Type II diabetes (lifestyle diabetes)


(1) Pancreatic B cells still able to produce some insulin just there is an insulin
secretory defect,
(2) Body cells do not recognize the insulin (insulin resistance)

Aetiology / risk factors: Multifactorial


- Obesity/overweight (80% diagnosed), age, family hx, periodontal disease (2 way
relationship), … …

…Body still has some insulin in circulation  isn’t breakdown of proteins/fats as seen in type I
diabetes ---> no ketoacidosis

Treatment: involving dietary control, possible medications (Metformin) to increase insulin,


blood glucose monitoring
Medication Treatments…
TYPE II
- Metformin – allows the body to use insulin more effectively
- Oral hypoglycaemic agents (OHAs) – Reduces HbA1c levels --> lower blood glucose
- Sulphonylureas (SU) – acts to increase insulin secretion from the pancreas, ineffective
after long use
- Insulin – very effective however does have many disadvantages (low therapeutic index,
inducing hypoglycaemia, difficult delivery injecting… … …)
o **Insulin pump delivered
- Thiazolidinedione (e.g. pioglitzone, and rosiglitazone)
- Acarbose

Diagnosis of diabetes

==Identifying patients with hyperglycaemia:


 Random plasma glucose level of > 11.1 mmol/l
 Fasting plasma glucose level of > 7.0 mmol/l

If reading detected along with other symptoms then there can be a definitive diagnosis made
Possible symptoms include:
-> Tiredness, Weightloss, Blurred vision, Thrush, Polyuria, Polydypsia, Nocturia

If these readings are detected in absence of other symptoms, then 2 readings showing incorrect glucose levels
should be detected for a diagnosis to be made

==Oral glucose tolerance test: Positive if >7.0mmol/l after 0 hours, AND >11.1mmol/l after 2 hours

Dental complication
 Dry mouth – due to polyuria
 Sialosis + sialadenosis (swelling of saliva glands)
 Tooth loss
 Gingivitis
 Periodontal disease more severe/prevalent
 Soft tissue lesions in the mouth and tongue
 Fungal infections – oral candidiasis

Diabetes and periodontal disease


 Bidirectional relationship
…Poorly control diabetes  increase risk of periodontal disease
(1) – Poorly controlled diabetes -->
  interleukin (lymphocyte activating factors) in GCF
 Compromised ability to respond to bacterial infection
  levels of systemic markers of inflammation

(2) – Compromised host response  proliferation of microflora

(3) - …Increased periodontal destruction caused by inflammatory mediators

…Periodontal disease  more difficult control over blood glucose


(1) – Periodontal pathogens and virulence factors enter the blood stream
--> Systemic inflammation
(2) – Leading to  decreased pancreatic B-cell function +apoptosis_
(3) – Reducing insulin control
Dental treatment for a patient with diabetes

Initial appointment- Determine patient’s normal routine/activity


- Ask patient to bring glucose monitor

Treatment - Check patient has followed normal routine on that day


…If they missed a meal reschedule, or send them to eat and return after 30 min
**DO NOT give glucose/sugary drink
- If the patient feels ill, cease treatment and assess blood glucose

**Appointment timing – mid morning or early afternoon

Delayed soft tissue healing – in patients with uncontrolled diabetes


- Assessment of a patients long term control of diabetes can be completed with HbA1c (glycated
haemoglobin) levels
o …measurement of average blood glucose conc. over previous 2-3 months
o Glucose attahces to RBS through the HbA1c, - glucose remains attached for RBD lifespan
(120days)
- Target score: = 7.0% (53mmol/mol)
- Delayed soft tissue healing: = 8.0% (64 mmol/mol)

Delayed healing, also places pt at increased infection risk

Patients on insulin : Assess blood glucose monitoring before treatment commences: SAFE LIMITS…
Random blood glucose between 3.5 – 12 mmol/l (normal 3-8)
 Proceed dental treatment

Random blood glucose > 12 mmol/l


 Refer to patients physician as medication needs to be altered

Random blood glucose < 3.5mmol/l OR patients exhibit signs of hypoglycemia


 Administer glucose and treat as medical emergency

Dentist’s impact – can induce fasting with sore mouths after treatment
 Remind of importance of regular meals
 Consume softer foods if necessary
o Administering local anesthetic ensure that the patient has
maintained normal routines and meals
Ensure  Pt condition is stable

Questions to ask:
 What type of diabetes do you have?
 How long had you had it for?
 Are you currently taking any medication for the diabetes? When was the last time you
took your medications?
 Did you eat before the appointment?
 What was your last known BGL / HBa1c reading?
MEDICAL EMERGENCY – HYPOGLYCAEMIC ATTACK
Occurring when BGL fall below 3.5 mmol/L, or low enough to exhibit signs
- Risk increased with abnormal routine, failure to take medication, excessive
exercise
Symptoms:
 Adrenergic (mediated by sympathetic nervous system)
o Pale skin
o Sweating
o Shaking
o Papitations
o Feeling of anxiety
 Neuroglycopenic (due to altered brain function
o Suboptimal intellectural function, confusion
o Hunger
o Coma
o Seizurees

IN INCIDENTS OF ATTACK:
Patient conscious + cooperative:
1) Cease dental treatment
2) Give 20-25g of glucose (or fast acting glucose food or drink = fruit juice, lemonade, jelly
beans, honey)
3) On recovery – provide low carbohydrate snack (sandwich, dried fuit)
4) Keep pt under supervision until fully recovered (do not allow them to drive home)

Patient unconscious + uncooperative


1) Cease dental treatement
2) Call 000
3) If patient is unconscious, commence basic life support
THYROID DISEASES Classed as ‘other’ med condition

Hypothyroidism (Hashimotos thyroitisis) - Autoimmune destruction of thyroid gland by T-cells

Symptoms: T3 and T4 involved with


- Energy loss /weakness metabolism
o Unexplained weight gain (body stimulating … decrease effects
hunger) energy, thermostat,
- Constipation gut motility…
- Anaemia
- Impaired immune function
- Sensitive to cold Dental implications – hyothyroidism
- Bradicardia - Risk to opportunistic infections
- CNS: tremors, behavioural change - Enlargement of the tongue
- Delayed tooth eruption
Treatment: Synthetic thyroid hormone

Hyperthyroidism (Grave’s Disease) - Additional antibodies in the blood driving thyroid hormone
levels up

Symptoms:
- Thyroid goiter
- Tremors / Palpitations
- Difficulty regulating temperature
- Sweating
- Stimulation of GI tract = diarrhea, feeling hot, weight loss (burning additional energy)
- Weight loss (generally)
- Opthalmopathy – swelling of eye muscles

Treatment: Carbimazole (blocks thyroid gland  time to reduce in size)


Surgery – to reduce gland size

Dental implications – hyperthyroidism

- Risk of THROID CRISIS – avoid LA with adrenaline (not contraindicated)


o General anaesthetics should be avoided
- Unstable condition
== Treatment should be deferred / avoid using LA containing adrenaline.
…risk of inducing a thyroid crisis / storm

Thyroid crisis / storm: Potentially fatal condition associated with untreated


hyperparathyroidism
 Elevated heart rates, BP and core body temperature
ADRENAL DISEASES

Hypofunction of adrenal glands (Addison’s disease) -Adrenocortical insufficiency – insufficiency of


secretion of ACTH by the pituitary gland
(…LOSS of Cortisol,
Aldosterone, androgen)
Possibly due to destruction (possibly autoimmune) of the adrenal cortex

Symptoms:
- Hyponaetramia
- Hyperkalaemia
- Hypoglycaemia
- Moderate acidosis
- Melanin pigmentation – hyperpigmentation
o White patches can also be found on the mucosa

Dental implications – hyperfunction of adrenal gland

Addisionian / adrenal crisis –pt unable to respond to stress due to low levels of cortisol from
dental appointment leading to possible cardiovascular collapse
 Occurring 6-12 hours after treatment  SCHEDULE MORNING APPT.
 Therefore if addionsian crisis it to occur they are awake
 Patient will feel faint, confused and collapse

 …day before procedure, pt should double their Corticosteroid dose
(assist in coping with stress)

Long term use of corticosteroids >3 weeks with 10ml per cay can lead to adrenal
suppression,

Treatment: (all require steroid cover – effecting treatment)


 Corticosteoids include…
 Glucocorticoids – involved in metabolism of carbohydrates/ proteins /fats
 Mineralcorticoids – assist in sodium and water regulation
--->Administration during times of stress

Hyperfunction of adrenal glands (Cushing’s disease)

Causes: Ectopic ACTH acting hormones (from tumors), hyperplasia of adrenal gland,
iatrogenically stimulated
…Increasing cortisol production

Symptoms: Dental implications – Cushings


- Hypertension disease
- Osteoporosis (inhibits bone formation) - Prone to bleeding (thinner
- Altered fat deposition – moon face, buffalo hump skin/mucosa)
- Muscle weakness at extremities (proteolysis) - Longer wound healing
- Depressive mood - Commonly oral candidiasis
- Skin thinning  easy bruising, straie on abdomen evident

Treatment:
- Removal of tumor, adrenal surgery, treat tumor associated with acting hormones
Conn’s disease: Tumor/hyperplasia of adrenal cortex (increased aldosterone production)

Symptoms:
- Hypertension, hypernaetramia
- Hyperkalameia – headaches, palpitations, polydipsia, polyuria)

Phaeochromocytoma – Increased secretion from the adrenal medulla due to a benign tumor
…palpitations, perspiration, hypertension

PIRUITARY DISEASES:

Hyperfunction: tumors of the pituitary glands:

 Pituitary adenoma (benign)(a secretion of hormones from glandular structures)


 Prolactinoma – excessive secretion of prolactin
o Symptoms include:
 Hypogonadodism – infertility,
 Amenorrhea, Galactorrhea (milk discharge), male breast enlargement

 Acromegaly – excessive secretion of growth hormone


o Results in the growth of many tissues:
 Skin / Connective tissue / Cartilage / Bone / Epithelial tissues… Etc etc
o Hypertension, diabetes, enlarged frontal/nasal bone
Oral manifestations – Acromeghaly
- Enlargemnt of the jaws
 spacing of teeth
 Particularly on the mandible causing Class III
- Enlargement of tongue

Hypoparathyroidism
Symptoms: - Associated with facial twitching, paraesthesia

Hyperparathyroidism: Rare – caused by adeonoma/hyperplasia of the parathyroid gland

Symptoms:
- Irreversible damage of the renal system
- Appears with ground glass appearance of bone
GASTROINTESTINAL DISEASES

Coeliac disease Gluten intolerance (hypersensitivity response of the small bowel to gluten) (long
term autoimmune disorder resultin in intestinal malabsoption)
- …therefore depression of immune system gluten drives the immune system
- T-cell immune response  small intestine immune damage (lacking villi for absorption
-> malabsorption)

Symptoms/manifestations (possibly asymptomatic)


--> Anemia - present with iron/foliate deficiency
--> Weight loss
--> Cramps + diarrhea following gluten ingestion (gluten intolerance)
BUT – don’t remove gluten from diet to dx, because without immune system doesn’t
function so may mask other dx.
--> Headaches/seizures
--> Infertility
Oral manifestations of Coeliac disease:
--> Osteoporosis
Aphthous ulcers (may be the only symptoms in adults)
Treatment Gluten free
- Glossitis, stomatitis, angular cheilitis
diet lifelong
-

**NOTE: different to irritable bowel –


Inflammatory bowel disease  Crohns disease
this is temporary and does not cause
 Ulcerative cholitis)
weight loss or bleeding in stool**
(1) - Crohns disease – Full thickness inflammation of the bowel mucosa, affecting any part of the
GI tract
…but generally lower portion of small, and upper portion of large
intestines (with skip lesions)
Aetiology – unknown

Symptoms: (can be evident elsewhere in other areas)


 Skin rashes, arthritis
 Fistulae can develop connecting organs together

Oral manifestations of Crohns disease:


- Oral ulcerations (tending to affect gingiva, buccal mucosa and lips)
- Lips: purple/red swellings
- Cobbledstoned buccal mucosa (with mucosal tags and folds)
- Orofacial granulomatosis
 Identical to clinical presentation of Crohns
 Possible other aetiology of hypersensitivity response to certain foods
– Soft tissue enlargement – facial or labial swelling
- Angular cheilitis
Treatment: Immunosuppression (dental implication), surgery

(2) - Ulcerative colitis Inflammation affecting only large intestine (responsible for water removal)

Symptoms: - Urgency for the toilet, bloody diarrhea, cramps Oral manifestations…
- Athritis Apthous ulcers
Treatment: - Specific gut anti-inflammatory drugs (and possible immunosuppression)
- Surgery (removal of the large bowel - mold small intestine to replace)
**CAN BE DIFFICULT TO DESTIGNUISH FROM CROHNS**
**CAN BE DIFFICULT
Behcet’s disease Severe relapsing and remitting oral and genital inflammation TO DESTIGNUISH
FROM CROHNS**
Characterized by: - Oral and genital ulcers
- Uveitis – inflammation of part of the eye
Oral manifestations…
- Skin lesions
Apthous ulcers
- GI involvement

Osler-weber-rendu syndrome Hereditary disease of skin causing abnormal formation of blood


vessels in the skin ( spider veins)

Symptoms: - Recurrent nose bleeds Can affect the lungs and


- Anaemia (iron deficiency) due to constant bleeding liver leading to shunting
- Melena and heart failure
- Signs of duodenal carcinoma – cancer of the duodenum
Oral manifestations of Osler-weber-rendu syndrome:
- Pigmented spots on the lips due to superficial
capillaries
-

Gastro-Oseophageal reflux disease (GORD) Transit of gastric contents into the oesophagus
**GORD is the term used to describe symtoms/signs

Normally – lower oesophageal sphincted, extrinsic compression by diaphragm, and pressure on


oesophagus prevent movement

Symptoms:
 Histopathalogical: Scarring of collagen tissue  constricting oesophagus to restricted flow
 Oesophagitis
o Damage of oesophagael epithelium with exposure to pepsin and acids
…at least half of reflux patients have oesophagitis
o Barrett’s disease –lining of oesophagus develops to similar to the stomach
**Premalignant – can become malignant (biopsy taken)
 Erosion of lingual or palatal surfaces of teeth

Diagnosis:
- PPI - Proton pump inhibitor test – (inhibits production of stomach acid)  symtoms stop
- Endoscopy with biopsy (taken for high risk pt)
- pH Monitoring (probe 5cm above LE sphincter

Treatment: (Symptoms may only be transient: 40% of people only 1/month no tx required)
- Conservative: (elevate bed head, smaller meals, weight loss, time between sleep+meals)
- Medical (medications):
o OTC H2A – for mild reflux, used when required
o PPI – superior to OTC
- Surgery (tighten top of stomach, reduce acid movement)
Malignancy is rare,
Peptic ulcer disease Ulcers of the gastrointestinal tract mucosa but peptic ulcers
…Due to acid (pepsin) degradation of epithelium may lead to
Aetiology: cancers
- Helicobacter pylori (infection – increases acid production causing ulcers)
- NSAIDs – inhibits prostaglandin (so reduces protecting mucosal lining)  Ulceration
o Must be taken with food
o …Consider pt past history when prescribing
 (Prevention with use of alternate medication, paracetamol)
- Concomitant used of corticosteroids and warfarin
Symptoms:
- Abdominal pain, Vomiting blood
- Melena (black stool)
- Fatigue

Bowel cancer

Risk factors: Ages, familial like, ulcerative colitis, smoking, other cancers

Adenoma-carcinoma sequence
...Most colorectal cancers a thought to arise from polyps Stage 0, 1, and 2 – still contained
(polyps progress through stages of dysplasia ----> Malignancy Stage 3 – in the lymph nodes
(spreading beyond basement membrane to blood vessels Stage 4 – in the liver

Symptoms:
 Rectal bleeding (anaemia)
 Altered bowel habits
 Abdominal pain (usually a late symptom)

Diagnosis: EARLY DETECTION - Faecal occult blood test (stool analysis) followed up by
colonoscopy (esp. >50)

Treatment: (polyp removal, chemo and radiotherapy)


Question 10: Have you ever had hepatitis, or any other liver disease?
DISEASES OF THE LIVER

The liver has dual blood supply, mainly venous blood for filtration of toxin and waste.
Function in metabolism of drugs, clotting factors production, bile production, glucose storage

 Acute liver failure …Can lead to liver failure


 Causes: paracetamol overdose (4g/day limit), viruses and other toxins

 Chronic liver failure (>6months) leads to cirrhosis **ALCOHOLIC LIVER DISEASE**


 Causes: alcohol, hepatitis B and C
 Liver Cirrhosis is the scarring (regeneration) of the liver, required greater pressure for blood
to pass through so blood will seek alternative routes

Symptoms of liver failure/disease:


- Loss of filtration  JAUNDICE (primary symptom – caused by inability of liver to process
bilirubin, the breakdown produce of haemaglobin; yellowing skin and eyes)
- Poor filtration  toxins remaining in the blood
- Drugs lack desired effect (buildup  toxicity)
- Poor blood clotting
- Alcohol related (chronic overconsumption of alcohol -> fatty liver, alcohol
hepatitis, and chronic hepatitis with cirrhosis) -> fatigue, nausea, weigh loss, (later: fluid in
legs, jaundice, abnormal bleeding
Dental: xerostomia, oral cancer, bruxism, parotid enlargement angular cheilitis, impaired healing, glossitis

Dental considerations for liver disease:


- Increased bleeding potential (additional local precautions required)
- Inablity to metabolize + excrete drugs
o Caution when prescribing drugs and administering LA (possible reduced dosages)
- Spread potential of hepatitis B / C / D
INFECTIOUS DISEASES Question 10: Have you ever had hepatitis, or any other liver disease?

HEPATITIS: Ask the patient = What type of liver disease? How long have you had it?

- Symptoms: Jaundice, abdominal pain/swelling, swelling in legs/ankles, dark urine color, pale
stool color, chronic fatigue, nausea and vomiting

Hepatitis A (Transmission –faecal-oral, contaminated foods)


- Jaundice common symptoms
- Treatment: Generally resolves itself in 1-2months

Hepatitis B – associated with liver diseases Hep B virus survives for a long time outside of
the body, and is highly contagious
Outcomes of hepatitis B infection:
 Acute hepatitis with clearance of the virus
- More likely if the hepatitis is more severe (bigger immune response)
- 5% require a liver transplant after liver failure

 Chronic hepatitis infection


- More common if infected young (infected young – more likely a carrier)
- Lifelong flares of the infection leading to liver cirrhosis

Symptoms:
 Anorexia, Nausea/vomiting Transmission – virtually all body secretions
 Abdominal discomfort …Blood borne: Body fluids
 Jaundice and dark urine – due to liver effects through the skin, skin piercings
 Asymptomatic (often in children) not sterilized, sharing needles

Treatment – No cure, about suppressing viral replication to prevent cirrhosis and cancer

Prevent infection: Vaccination, correct standard precautions, post exposure prophylaxis, condom use

Hepatitis C – associated with liver diseases RNA virus with no vaccine, blood borne virus
Dental implications… higher risk of dental
Symptoms: - Lethargy
caries and periodontal disease (decreased
- Loss of appetite, Nausea and vomiting
salivary flow)
- Fever and pain in joints
 heavy emphasis on prevention
Treatment – curative (can eradicate because it involves RNA) (can cure most cases 80%)

Dental implications of ALL hepatitis infections


- Dental staff  All members of dental staff should be vaccinated against
- Standard precautions followed with all procedures
o Sharps injury risk -> may cause transmission (take blood test to confirm)
-  drug metabolism   risk of drug toxicity and hepatotoxicity
o LA and sedatives reduced dosage
o NSAIDs avoided – Can cause GIT bleeding -> renal impariement
 Paracetamol is drug of choice
- Consider AP prophylaxis requirement
 For patients with advanced Hep C
Question 11: Do you currently have, or have you recently been exposed to an infectious disease?

HIV/AIDS – Viral infection attacking the body’s immune system, namely the CD4 (T-helper) cells,
decreasing the body’s ability to combat simple infections

-- AIDS (acquired immunodeficiency syndrome) is the 'disease' (opportunistic


infections that overcome the immune system) (terminal stage of infection)
-- HIV (human immunodeficiency virus) is the virus that may cause AIDS

Transmission – Sexually, blood to blood, mother to baby

Symptoms: Dental implications of HIV


 Fever - Antiretroviral drugs (ALL pt taking this) interact
 Headaches with many commonly prescribed drugs
 Lethargy (erythromycin, codeine, itraconazole, diazepam)
- Patients at risk of infection immunocomromised –
 Nausea/vomiting/diarrhea
consider AB prophylaxis
 Weight loss
- Possible bleeding risk
Oral manifestations: - Needle stick injury – PEP within 24 hours
- Strongly associated:
o Oral candidiasis, angular cheilitis
o Hairy leukoplakia (bilateral, white, non-removable corrugated lesions of the
tongue)
o HIV gingivitis/periodontitis (severe even with good oral hygiene)
o ANUG
o Kaposi’s sarcoma (one or more erythematous/purplish macules/swellings) –
malignant potential
o Non-hodkins lymphoma
o Salivary gland enlargement
- Others less strongly associated, and possibly associated

What to do in the case of Needle-stick Injury?

1. Immediately – wash skin with soap/alcohol (no squeezing)


2. Assess the risk – exposure type, amount of fluid involved, infectious state of pt.
3. Test patient and HCW (test source for hep B / C / HIV, test HCW for baseline serum
NOTE – If patient is HBV/HCV/HIV negative, then no follow up is required.

4. Possible treatment if patient is positive for:


a. HBV vaccinated within 24 hours (or Hep B antibodies within 24 hours to
immunocompromised)
b. HCV  RNS testing after 4-6 weeks, antibody tesing after 4-6 months
c. HIV  HIV antibody testing 6 weeks, 3 months, and 6 months after exposure
 If post-exposure prophylaxis is indicated, commence a 28-day course as soon as
possible with 2-3 anti-retrovirals
5. Counselling and follow up
NOTE – There is a balance that must be considered between giving a healthy person a potentially toxic
compound with side effects (prophylactic anti-viral), and the perceived reduction in risk of infection
Question 8: Have you ever had tuberculosis, asthma, or any other lung diseases?

Tuberculosis Lower respiratory tract infection cause by bacterium: mycobacterium TB


…Increasing incidents
…Transmission through airborne droplets

Symptoms: cough/sweats/weight loss/ fever/ couching up blood/

Treatment: long treatments (6-12 months), isolation in negative pressure room


Oral manifestations of tuberculosis
Patient free of active disease  treated as normal

- Infection control:
--DEFER PT WITH ACTIVE DISEASE if possible
… Dental Treatment of patients must use submicron masks
…Completed in negative pressure room (hospital)
- Oral manifestations:
…Boney radiolucency
…Ulceration (deep / irregular / painful) on tongue dorsum
-- Can also effect palate, lips, gingiva

==> Rifampin = medication can delay healing, and cause ginginval bleeding

Question 12: Have you ever undergone neurosurgery (prior


to 1982) or growth hormone treatment (prior to 1985)?

History of neurosurgery or growth hormone treatment


 Neurosurgery prior to 1982
o Surgeons used to graft the dura mater from human remains (high infectivity)
 Growth Hormone prior to 1985
o Surgeons used to extract hormones from pituitary glands (high infectivity)
 Dental implications:
o These individuals may be pre-disposed/carriers of CJD
o Use of disposable instruments
 Sterilisation process ineffective in removing prions

Question 13: Do you or any members of your family have a history of Creutzfeldt-Jakob Disease (CJD)?

Creutzfeldt Jacobs disease Invariably fatal human prion disease (resulting in pathological
accumulation in the brain of the prion protein causing cell death in
Mad-cow Disease
the brain)

Transmission: - Sporadic, inherited


- IATROGENIC TRANSMISSION (through neurosurgical instruments)
…Extremely rare transmission (epidemiology = 1/1,000,000)
…Need to use disposable instruments if pt is a carrier of CJD
…Prion prelated – steralisation does not remove prions

Symptoms: Cognitive impairment, memory loss, death


MUSCULOSKELETAL DISORDERS
Osteoporosis Imbalance between osteoclast and osteoblast function resulting in a
demineralization of bone structure (decreased bone density)
…Making bones fragile and susceptible to fracture

Prevalence: More common in women **Osteopenia = bone mass lower than normal, but
Causes: - Oestrogen deficiency (role in not low enough to be classified as osteoporosis**
osteoclast+osteoblast function)
- Vit. D deficiency
Dental implications of osteoporosis
- Prolonged immobility
Direct: -> Loss of alveolar ridge 
- Chronic diseases (endocrine, malnutrition…)
periodontal disease
- Malabsoptions (inflammatory bowel, coeliac)
-> Bone resorption in edentuolous
- Iatrogenic (adverse drug effect, radiotherapy)
patients
Symptoms
-> Tooth loss
 Reduced height, Kyphosis (stooped posture)
-> Jaw fracturing
 High fracture risk
 High mortality rate (from hip fractures in old age)
Treatment implications:
-> Bisphosphonates  BRONJ ****
Treatment: -> Calcium alters the absorption of
- Prevention (lifestyle factors – exercise, diet, vit. D)
antibiotics
- Medications (calcium and vitamin D replacement,
-> Patient positioning
denusomab)
 Bisphosphonates (encourage
building of bone)

Paget’s disease – osteitis deformans Normal orderly replacement of bone is disrupted and instead
replaced by a chaotic structure of new bone
…the bone is thicker but weaker
Symptoms/complications:
- Large effects in skull, Dental implications of Paget’s
spine, pelvis and legs Direct:
- Secondary arthritis  Root resorption  tooth loss and periodontal disease
common  Bone loss at the roots  tooth mobility and loss
 Jaw involvement (mainly the maxilla) (boney enlargement
 Bone growth – malocclusions, teeth spreading, hypercementosis

Treatment implications:
 Bisphosphonates leading to BRONJ

Question 16: Do you have any Joint problems, Arthritis, or history of joint replacement surgery?
Osteoathritis Arthritis due to general wear and tear on joints (degradation of articular cartilage)
*Questions = what joints affected? Impede ability for oral hygiene?
Dental implications of osteoarthritis
Causes: Wear and tear (age + Direct: -- Manual dexterity reduced – difficulty with oral hygiene
overuse – greater risk with consider electic brushes, modified handles
obesity) --TMJ possibly involved (unilateral, pain, decreased
opening, swelling, sounds, tenderness)
Symptoms: Pain, stiffness of joints -Consider chair time + pt. comfort
Treatment implications:
Treatment: --Possible opiod use  side effect of dry mouth
- Analgesics to suppress the pain -- Medication – NSAIDs
- Surgery for joint replacement Dental treatment considerations:
AB prophylaxis not indicated UNLESS previous. Current hx
INFLAMMATORY ATHRITIS…

Rheumatoid arthritis: Chronic inflammatory disease resulting in autoimmune


destruction of the synovial membrane of a joint
*Questions = what joints affected? Impede ability for oral hygeine? Difficulty supine?

Symptoms:
- Painful joint swellings, warm
o Usually symmetrical (especially hands, feet and knees), (immobility)
o Particularly in the immobility
Dental implications of rheumatoid arthritis
Treatment:
Direct:
- Physical therapy
- NSAIDs  TMJ involvement (RARE)
- DMARDs  Manual dexterity reduced – difficulties completing oral hygiene
(methotrexate – Treatment implications:
interacts with  Corticosteroid (effect immune system  infection risk)
NSAIDs)  Interactions with NSAIDs, corticosteroids, immunosuppresants
- Analgesics to Dental treatment considerations:
relieve pain  Chair time +Mobility /flexibility of patient –access difficulties
- Corticosteroids  AB prophylaxis with recent joint replacement (past 3 months)
- Surgery

Gout / Crystal arthritis Urates deposited in joints


^^Pseudogout^^
Symptoms:
- Affects any joint but mainly large joints , commonly big toe
- Severe pain
Dental implications of Gout
- Joint red, inflamed, very tender
- Drug adverse effects
o NSAIDs
Treatment:
o Probenecid – sore gums
- Lifestyle factors
o Allopurinol – altered taste, oral parasenthia)
- Medications (NSAIDs,
corticosteroids, probenecid,
colchicine, allopurinol)


 Hereditory heamocromatosis

Reactive arthritis syndrome (Reiter Syndrome): Seronegative arthritis, urethritis, conjunctivitis


…often response to infection
Dental implications of Seronegative arthritis
Direct:
 Oral lesions often present: Oral burning + Gingival erythema (redness)
 White patches of oral lesions
Arthroplasty (Joint replacement surgery)

“Current literature does not support use of prophylactic antibiotics for all patients with prosthetic joints”

 Definition:
o Surgical replacement or reconstruction of a joint using prostheses
 Indications:
o Severe arthritis
 Osteoarthritis
 Rheumatoid arthritis
o Trauma
o Misaligned joint
o Many other reasons
 Dental Significance:
o Before placement: - Patient rendered orally fit for treatment
o After placement:
 Small risk of infection at the prosthetic site by the haematogenous route
 AP not recommended – consider risk vs benefits
 Antibiotic toxicity / Allergic reaction / Microbial resistance
 Dental problem within 3 months following artificial joint replacement:
 Infection with abscess = remove cause, treat aggressively, No AP
 Pain = emergency dental treatment for the pain – AP
 Non-infective dental problem without pain – defer dental treatment
until 3-6 months after prosthesis replacement
 Dental treatment after 3 months:
 If normal functioning artificial joint = No AP
 Dental treatment for pt. with significant risk factors for artificial joint infection:
 Immunocompromised = diabetic, medication, rheumatoid
 Non-essential treatment = defer until immunity has stabilized
 Essential treatment = consult with orthopaedic surgeon – usually procced
with AP
o AP indicated:
 Previous history of artificial joint infection
 Established infection of joint
 To eliminate any oral cause
CONNECTIVE TISSUE DISEASES …

Sjogren’s syndrome: Triad of xerostomia, dry eyes (conjunctivitis) and connective tissue disorder
(usually rheumatoid arthritis)

…Due to the infiltration of immune complexes that run into salivary and lacrimal glands
resulting in destruction/fibrosis
…Autoimmune aetiology Dental implications of Sjogren’s syndrome
Direct:
Symptoms: (**triad**)  Xerostomia
- Xerostomia  Glossophyrosis (burning tongue)
- Dry eyes  Tongue and lip fissuring
- Connective tissue disease 
 Candidiasis
causing rheumatoid arthritis
Indirect
 Parotid swellings (uncommon)
Treatment:
 Difficulty chewing/biting/opening the mouth
- Treating the symptoms: Occular
Treatment implications:
lubricants, Salivary stimulants and
substitutes  Drugs adverse effects (immunosuppression,
- Drugs: NSAIDs, DMARDs (disease ulceration)
modifying anti-rheumatic drugs)

Ankylosing spondylitis: Spine becomes stiff and inflexible (ossification of the spinal ligaments)
Symptoms:
- Bamboo spine Dental implications of Ankylosing spondylitis
Treatment: - Changes to TMJ
- Stretching, analgaesia, surgery - Mobility and access difficulties

Lupus erythematosus Autoimmune inflammatory disease causing red patches on the skin
Causes oral lesions: Orofacial eruption (red rashes), lesions of lips, white ulcerations, bleeding
after extraction, secondary sjrogens syndrome

Osteomyelitis Bone infection causing inflammation of the bone marrow


…Commonly staphylococcus aureus

Aetiology: Infection from root canals, periodntal pockets, extraction sockets


Radiological changes
Treatment: Long term antibiotics, surgical debridement evident with osteomyelitis

Osteogenesis imperfect: Genetic defect in collagen formation causing


dentinogenesis imperfecta
Brittle bone disease
Dentinogenesis imperfeta:  affected teeth brown/blue hue,
bulbous crowns, short roots

Achondroplasia Dwarfism, causing malocclusions (class III) and skull bossing

Cleidocranial dysostosis Affecting skull and clavicles, Persistent deciduous dentitions


**Medications associated with many musculoskeletal disorders**
 NSAIDs  Opiods  Paracetamol  Corticosteroids

 Bisphosphonates

Bisphosphonate related osteonecrosis of the jaw - BRONJ:


- Osetonecrosis: Death of exposed bone following surgery or traumatic injury
- An area of exposed bone in the jaws persisting for more than 8 weeks
…DDX= rule out osteoradionecrosis (pt has hx of radiotherapy of the head/neck)
Symptoms:
- Painful
- Exposed bone visible
- Possibly associated with draining sinus, or soft tissue infection (can be extensive)

Aetiology:
- Bisphosphonates reduce the osteoclastic bone resoption ( reduce bone turnover so there
is growth of bone)
- BRONJ---
o ---Most commonly following tooth extraction
o …possibly associated with poor fitting dentures

Classifications:
- STAGE 0 = No exposed bone but bone pain
- STAGE 1 = Exposed bone but no pain
- STAGE 2 = Painful exposure of bone with soft tissue/bone inflammation
- STAGE 3 = Full thickness bone involvement, pathological fracture, extensive soft tissue
infection

(1) Patient dentally fit before commencing bisphosphonate treatment


- Undertake comprehensive oral examination to ensure dentally fit, unlikely to require exo
o Complete required tx to get the pt to this stage

(2) After commencing bisphonsphonates – regular monitoring or oral health


- Treatment required should be inercenpted with concervative measures (avoid exo)

Patients on bisphosphonates requiring extractions:


- Do not complete extraction until we have determined risk of BRONJ
- ASSESSEMNT OF C-TERMINAL TELOPEPTIDE (CTX) in serum Low CTX
o Breakdown product of bone resportion (provides estimate ==> Low breakdown
of bone turnover) ==> formation of bone
 Normal: 400-500 pg/mL ==> BRONJ Risk
 Negligible risk = Able to proceed: >150pg/mL
 Able to complete bone invasive proceedures safely
 Moderate risk: 70-150 pg/mL
 High risk: <70 pg/mL

o ****<150pg/mL consider drug holiday


 Cease bisphosphonates, CTX increases 25pg/mL per month

**If extraction is unavoidable – it should be proceeded with minimal trauma and suturing
**BRONJ can also occur with Bisphosphonate alternatives = e.g Denusomab
RENAL DISEASES Question 15: Have you ever had any kidney problems?

Afferent network ------> Efferent network


Glomerulus (filtrate produced)
---> Proximal convoluted tubule
---> Loop of henle
---> Distal convoluted tubule
---> Collecting duct

Kidneys are involved in the regulation of blood pressure and stimulation of blood production
…Renin-angiotensin-aldosterone mechanism …EPO release

Confirmation of dx usually completed with urine analysis for


 Proteinuria (pregnancy, diabetes, infection),
 Glucosuria (diabetes, infection, nephrotic syndrome),
 Haematuria (RBC, WBC – infection, tumor, kidney stones, glomerulonephritis)

 Nephrotic syndrome
- Allows for proteins to pass through the urine (Proteinuria)
- Caused by glomerulophephritis
o  generalized oedema and facial oedoema

 Nephritis syndrome
- Allows RBC to pass into urine (Haematuria)
- Caused by glomerulophephritis
o  generalized oedema and facial oedoema

Renal calculi - Kidney stones

Forms …Calcium containing / Uric acid based

Symptoms
 Very painful (if small and able to pass/get stuck in the urethra)
 Larger do not pass to the urethra
 Calcium based visible radiogrpahically (radiopaque), uric acid containing not visible

Treatment - Preventing formation – increased water consumption


- Pain relief
- Surgically removed

Glomerulonephritis Inflammatory disease of the kidneys


Causes: Mostly resultant of deposition of immune complexes at glomerulus filtration membrane
Decrease flow in glomerulus  water and sodium retention 
increased in blood volume  hypertension
*Types*- Many different types

Treatment: Often resultant of the immune system is in overdrive


 Suppress the immune system
--> Immunosupressants
--> Steroids
…Identify and correct precipitant if possible
Acute kidney injury (Acute renal failure) A medical emergency causing rapid rise in serum
creatinine, urea, and potassium

Acute renal failure can occur due to events prerenal, renal, or post renal

Causes…
 Prerenal => Hypotension  decreased flow through the kidneys
=> Hypovolaemia
=> Shock
 Renal system: => Glomerulonephritis
=> Nephritis (commonly due to drug allergy)
=> Malignant hypertention
 Post renal: =>Obstruction (prevents kidney drainage

Treatment: dialysis

Chronic renal failure

Causes: Due to a falling glomerulus filtration rate… *due to Glomerulonephritis, diabetes,


obstructions, polycystic kidney disease,
pylreonephritis (small scarred kidneys)*
Symptoms: - Uremia (due to falling GFR)
- Vomiting, Lethargy (nocturea interrupting sleeping patterns)
- Pale, itchy
- Oedema – fluid retention as filtration isn’t occurring correctly
- Confusion

Treatment: --> Symptom relief


--> Control blood pressure
--> Dialysis
Haemodyalisis – blood filtered by machine
IV access vital
Fistula required – DO NOT TOUCH THIS ARM
--> Transplant - immunosuppression (risk of opportunistic infections)

Dental implications of renal disorders:


 Increased risk of infection (particularly with immunosuppression
 Increased bleeding tendency
 Decreased ability to excrete drugs – consider dosages
 One lesions of the jaw
 Potential carrier of Hep B/HIV

Medications
 Corticosteroids - Tx with oral prednisolone
o >10mg daily for 3 weeks may induce adrenal suppression  addisonian crisis (to
counter this, dose of corticosteroids should be increased prior to appointment)
 Immunosuppressants (Cyclosporine  gingival hyperplasia)
 Dialysis (delay tx at least 4 hours post dialysis, ideally next day – Increased bleeding risk)
 Drug excretion
o Reduced excretion – liaise with GP and revisit drug dosages
 Drug metabolism
o Pt intolerance to drugs metabolised in kidney
o NSAIDs avoided for those with mild renal impairment ( water retention) –
paracetamol the drug of choice
 Abnormal bleeding ( erythropoietin production/ platelet dysfunction)
o Careful surgical technique/local haemostatic measures
 Adrenaline containing LA / Suturing
 Oral manifestations
| Mucosal pallor, pigmentation | Xerostomia | Parotid infection | Dysgeusia | Metallic taste
| Oral candidiasis | Enamel hypoplasia | Stomatitis | Loss of lamina dura |

*****Anaemia***** See hameatological disorders


…can result due to failure of erythropoietin production by the kidney
NEUROLOGICAL DISORDERS

Motor neuron damage/lesions **Patterns to the symptoms provide evidence of


if the upper or lower motor neuron is damage**
Upper motor neuron lesion
(Intact lower motor neuron)
 Nerves could still fire but not getting the stimulus to
 The muscle remains intact

**Pseudo Bulbular palsy**


 Weak but No/little muscle wasting
Upper motor neuron lesion, bilaterally affecting
 Spasticity (altered muscle action/paralysis)
the pons
 Hyperreflexia
-> (Possible causes of stroke and MS)
 Effecting groups of muscles …Swallowing and speech difficulties

Lower motor neuron lesion (e.g. motor neuron disease, resultant of diabetes, bells palsy)
 Weak with Muscle wasting – muscle receive no stimulus
 Reduced reflexes
** Bulbular palsy**
 Fasciculation (twitching of muscles)
Lesion of the brainstem nuclei (lower motor neuron),
 Effecting single muscles
effecting the tongue and mastication muscles
...Tongue flaccid, speech hoarse

**Bells palsy**
Inflammation of CNVII, response to viral infection
(Demylinisation)
…Facial paralysis: Unilateral sagging of
mouth and eye, dribbling, taste impairment
HEADACHES
 Severe acute
-- Meningitis (CNS infection and inflammation of the lining of the brain)
-- Head injury (possible bleeding)
-- Sinusitis – inflammation within sinuses (often infection related

 Acute recurrent
-- Migraine – distinct entity characterized by a preceding visual aura and severe,
unilateral headache
 Due to abnormal brain blood supply
 Symptoms:
o Severe, unilateral headache, photophobia, recurrent, nausea +
vomiting
 Treatment:
o Quiet, dark room (avoid stimuli)
o Medications: NSAIDs, paracetamol

-- Cluster headaches – Recurring headache causing localized pain (usually universal


in symptoms)
 Rarer than migraines
 Localized pain, usually around the eye
 Recurring for several weeks (usually felt in the morning)
 Sub-acute
-- Temporal arteritis/Cranial arteritis (Giant cell arteritis)–
Connective tissue disease often presenting with jaw pain, +
unilateral throbbing headache
 Major complication: optic nerve ischaemia  causing blindness
o  Emergency commencing medications (high-doses of steroids/
corticosteroids)
 …Prednisolone to prevent blindness
 Chronic
-- Tension headache (worse ending the day)
 MOST COMMON HEADACHE
 Muscle tension in occipitofontalis
 Getting progressively worse during the day
 Tx: reassurance, anxiolytics, analgaesics

-- Raised intracranial pressure (worse at the start of the day)


 Headache requiring further investigation
 Worse on waling, vomiting, loss of consciousness, ……
…others…
 Cervicogenic
 Occipital neuralgia
 Substance withdrawal
 Medication misuse headache – excessive/regular use of over the counter analgaesics
 Icepick – trigeminal nerve, one of the nerve branches abnormally fires
 Acute glaucoma – acute pressure in the eye
 Angina

OPM – classifies
neuropathic  Trigeminal neuralgia Sudden, usually unilateral, severe, brief, stabbing, recurrent
pain sensation of pain distributed in one or more branches of CN V
Aetiology:
- Compression of a root of CN 5 by:
o Blood vessel / tumor / MS plaque
- Can be secondary as a result of multiple sclerosis or post-herpetic neuralgia

Diagnosis / Clinical features:


 Pain history
o Character of pain: Sharp/electric/shooting/unilateral Pain lasting more than a
 Severity: excruciating 11/10 (often patients few minutes more likely
have suicidal thoughts) considered alternative
o Duration: 25 seconds with background dull ache facial pain
remaining (few seconds to <2minutes)
o Remission: Periods of latency where touching doesn’t cause pain
o TRIVIAL activity Exacerbating/causing pain: brushing teeth / shaving / wind
 **May be stimulated during dental exam - possibly trigger point on gingiva**

Treatment:
- Medications (primary treatment):
o Carbamazepine – sodium channel blocker / anticonvulsant stopping nerves from firing
**Effect on warfarin -> lower BP**
- General chronic pain management:
o Mindfulness based stress reduction
o Exercise and sleep hygiene
- Chemical/radio destruction of nerve
 Glossopharyngeal neuralgia Sudden/brief/severe pain in the glossopharyngeal nerve
…base of tongue …angle of jaw …tonsils
…^Much less common
…Pain on swallowing
- Treatment: carbamazepine, may require surgery

 Temporomandibular dysfunction (TMD)


Group of musculoskeletal and musculoligamentous conditions associated
with the TMJ, muscles of mastication, and associated tissues
Aetiology:
- Idiopathic / Multifactorial - complex interactions making true aetiology difficult to determine:
o Psychosocial features
 Stress induced parafunctional habits - chronic stress (anxiety, depression)
o Joint and muscle trauma
o Anatomical / Pathophysiological features
 Articular disorders – congenital disorders, disk derangements, dislocations,
inflammatory condition, arthtritis, ankylosis, fractures
 Muscle disorders – myofascial pain, myospasm, local myalgia

Clinical features:
 Unilateral / bilateral facial pain
o Trismus
o Joint clicking / crepitis / locking / pain / deviations
 Pain in muscles of mastication / TMJ
o Tenderness of temporalis / masseter / TMJ
3 levels of progression of TMD:
with palpation
 Subclinical features without
o Pain on chewing / opening wide
problematic features
o Inability to open wide
 Transient TMD features
o Pain in lateral movements
(occasional symptoms bouts)
 Headaches (of frontal, parietal, occipital region),
 Persistent TMD with
particularly on waking continual problems
 Ear pain / tinnitus (ringing in the ears)
 Eye pain

Diagnosis: - TMD is often inappropriately diagnosed, causing undue concern to the patient
- Adequate diagnosis based upon thorough history, examination and imaging

- OPG radiograph required


o Possible CT scans / MRI (relatively inaccurate)
- Blood test – deficiency in vitamin B12 may lead to TMD
- **Clinical features – tenderness of temporalis/masseter/TMJ, limited opening, pain on lateral
movements**

 Tooth pain (due to caries, inflammation of the pulp)


Blackouts

Numerous causes (hx to determine)


Differential diagnosis
 Epilepsy  Syncope  Strokes  Hypoglycaemia  Cough syncope

Seizures Usually due to a cause of abnormal biochemical status…


e.g. trauma/strokes/hypoglycaemia/hypoxia … … …

Question 7: Have you ever had a stroke, fits or epilepsy?

Epilepsy Group of neurological conditions characterized by episodes of recurrent seizures

…With altered consciousness, movement (tonic-clonic convulsions),


cyanotic, possible incontinence, facial trauma, biting the tongue
…Fits rarely lasting >5min

Generalized (tend to be complex) or Partial … Simple or Complex

Treatment: Carbamazepine, Na valproate


Dental implications of epilepsy:
When treating a patient:
1) Determine stability of epilepsy (determin possible triggers relevant)
2) Has the patient taken medication
…Person with epilepsy at risk of seizure if not taken medication in 48 hours

Completing dental proceedures:


 Avoid stressful, extended proceedures
 Use of a bite block
 Some medication induces gingival hyperplasia (good oral hygiene minimizes)

MEDICAL EMERGENCY – SIEZURE:


Prodromal symptoms – aura often precedes (cease dental tx and assess)
 Cease dental treatment
 Protect the patients from falling from the cair and injuring themselves
 Avoid restraining the patient (unless essential to avoid injury)
 Wait until fit subsides
 Assess consciousness
 Maintain airway
 If patient vomits, remove with high flow suction

…Patient recovers completely:


 Hold for 30 minutes before discharging.
Do not allow to drive home.
Advise to seek medical attention.

…**If the seizure lasts for more than a few minutes, or if there are recurrent seizures
without recovery between attacks (status eliepticus):
 Call 000
Maintain aiways
Monitor the patient until assistance arrives
Question 7: Have you ever had a stroke, fits or epilepsy?
Stroke Cerebrovascular accident  sudden loss of blood supply to the brain
(cerebral ischemia / hemorrhaging), leading to permanent damage
(necrosis) of the brain resulting in disability

Reduced blood flow with: Cerebral ischemia (clot lodging), or Hemorrhaging


Prevention of high blood pressure,
Prevention: and clot/thrombosis formation
(Modifiable) Risk factors: - Primary prevention – manage the risk factors
- Cardiovascular diseases: o Correction of modifiable risk factors
o Atrial fibrillation o Smoking cessation, diabetes control
o Hypertension o Control AF (warfarin)
o Hypercholestrolaemia o CV diseases – lower BP (aspirin)
- Smoking - Secondary prevention
- Diabetes o Very ^similar^ BUT additionally
involving antiplatelet therapy
(decrease blood clotting)
<<Hemorrhagic stroke – 1/3 of strokes are involved with bleeding

Transient ischemic attack (TIA) – lasting less than 24 hours with reversible damage>>
…transient, obstruction of the blood vessel (acute presentation)

Dental implications of stroke:


- Defer elective dental treatment 6 months after attack
- Motor defect of the arm  Impeded ability for Oral hygiene
o Possibly large handled tooth brushes, or electric brushes, are suitable
- Motor defect of facial muscles:
o Difficulty wearing dentures (possible modification) – may have food build on one ide
of the mouth
- Anticoagulant and antiplatelet medications
o Bleeding risk and impaired wound healing -> local haemostatic measures
o INR assessment if on warfarin (prothrombin time)

Signs/symptoms of a stroke / TIA occurring…


 Weakness, numbness, paralysis of the face/arm/leg
on one (sometimes both) sides of the body
 Difficulty speaking/understanding
 Dizziness, loss of balance
 Loss of vision, sudden blurred or decreased vision
in one (or both) eyes
 Headache (usually severe, sudden onset)
 Difficult swallowing

MEDICAL EMERGENCY - STROKE:


Check with F.A.S.T. principals
- Cease dental treatment
--Face – Check the face, mouth dropping
- Call 000
--Arms – can both arms be lifted
- Administer oxygen
--Speech – speech slurred, understanding you?
- Maintain airway
--Time – critical – Call 000 on any of these signs
- Monitor vital signs until assistance arise
**DO NOT give aspirin – difficult to determine if ishaemic or haemorhagic**
Multiple sclerosis Chronic demylinisation of neurons due to autoimmune destruction

Classifications:
Progressive relapsing - Steady decline with periods of attack and relapse
Relapsing-remitting - Attacks followed by remission (most common)
Primary-progressive - Slow progression (no attack or remission)
Secondary progressive - Progression more quickly, may experience attack/remission

Symptoms: - Numbness/tingling
- Weakness
- Dizziness
- Visual loss
- Fatigue
- Bladder/bowel/sexual dysfunction

Treatment:
 = Prednisalone – anti-inflammatory effect, adrenal crisis risk from dental tx (
dose)
 Interferon beta 1a or 1b – slows progression (reducing frequency of attacks)
 Significant side effects – cheilitis, xerostomia, gingivitis, candidiasis
 Natalizumab – limits entry of Tcells to the brain
 Fingolimod – alters lymphocyte migration

Parkinson’s disease Degeneration of substantia nigra dopaminergic cells


…Area of the brain responsible for dopamine production

TRAP symptoms > Tremor (T.R.A.P)


> Ridgitiy
> Akinesia – lack of stimulating movement
> Postural instability
++ -> Mask like face (limited emotions)
-> Shuffling gait
-> Dyskinesia – uncontrolled movements, a result of medications
-> Dementia – possible medication induced
-> Hypersalivation (difficulty visually in treatment)
+++ > Bradykinesia – slowness of movement

Treatment: = Dopamine agonists, leva dopa, amantadine

Dental Implications:
 Difficult in OH
 Excessive salivation and drooling
 Patient may have tremor and find difficulty
cooperating during operative treatment.
 Motor reflexes impaired – may be in a wheelchair
PSYCHIATRIC DISORDERS

Anxiety disorders Characterized by the emotion of anxiety, worrisome thoughts, avoidance


behavior, and somatic symptoms

Types: - Phobias (of specific object)


- Panic disorder (repeated attacks, not specific to object)
- Generalized anxiety disorder (Chronic anxiety)

Symptoms: Tachycardia, dizziness, hyperventilation, diarrhea, aggression, lack of concentration,


reduced pain threshold, dry mouth, bruxism

Treatment:
- Cognitive behavioral management (therapy – e.g. anger management)
- Counseling
- Medication provision (beta blockers, antidepressants, sedatives)

Dental relevance of anxiety disorders


 Patient may be aggressive, standoffish, unfriendly, sensitive
 Dry mouth – resultant of anxiety directly, and medication side effects
 Symptoms of hypoglycaemic attack (diabetic event usually tachycardia with paleness and sweating)

Dental treatment of the anxious patient…


1) Communication (sympathise, reassure, explain)
2) Sedation
3) Relaxation techniques (distraction)

Mood disorders Depression - continued depressive mood,(significant enough to effect everyday life)
Bipolar - periods of mania followed by depression, (significant enough to effect everyday life)
Aetiology/causes/triggers:
- Genetic predisposition (possible serotonin transporter gene involvement)
- Emotional deprivation in early life, Stressful life events (trigger)
- Side effects of medication (trigger)
- Secondary to pathological processes (e.g. endocrine) (trigger)

Symptoms: - Anhedonia (loss of enjoyment), weight change (reduced/increased eating), anorexia,


- Depressive mood, low self esteem, guilt,
- Suicidal thoughts (major cause of self harm),
- Diminished adherence to medical tx and rehabilitation

Treatment:
- Counseling | Psychotherapy | Cognitive behavioral management
- Pharmacologically treatment: Aimed to elevate serotonin and or noradrenaline
 Selective serotonin reuptake inhibitors (SSRI) + SNRIs
 Tricyclic antidepressants (TCA)
 Monoamine oxidase inhibitors (MOAIs) **In bipolar – depressive elements
a treated with antidepressant
medication, mania treated with
Dental implications of mood disorders:
antipsychotic drugs**
 LA interactions:
o Not contraindicated with SSRIs or TCAs
o MAOI avoided
 Issues gaining informed consent
 Xerostomia and dry mouth due to disorder and medications
 Increased incidence of TMJ pain and atypical facial pain
Dental implications of somatoform disorders
Somatoform disorders Somatic symptoms not explained  Inappropriate requests for cosmetic dentistry
 Burning mouth
by a medical condition and not  TMJ pain dysfuction syndrome (TMJPDS)
better diagnosed by depressive or  Parensthia - strange sensation (possibly
anxiety mood disorders lacking a cause)
 Atypical facial pain more common
Schizophrenia Delusion, hallucinations and lack of insight

Aetiology - Developed at birth, triggered later in life (commination)


^^Genetic component ^^Environmental risk factors

Dental implications of Schizophrenia:


Acute symptoms:
 Medications causing dry mouth
 Disturbed behavior
 Uncontrolled facial movement (dyskinesia) - difficulty in
 Delusions / Hallucination
treatment
/ Disordered thinking
o Acute dystonia (involuntary) muscle spasms 
 Social withdrawal broken teeth
 Bruxism
 Impaired pain perception (may be hypersensitive or lack
sensitivity)
Substance related disorders
 Gross neglect of the dentition
1) Alcohol related disorders
Effects:
 GI / liver / cardiac disease, CNS diseases
 Weight gain
 Psychological impacts (mood disorders, anxiety, self harm (trauma incidents,
psychosis)
 Deterioration of relationships
- Increased criminal activity (possibly violent toward the dentist)
- Work and financial issues (issues in economic access to treatment)

Dental implications of alcohol related disorders:


 Increased risk of oral cancer
 Poor compliance to treatment
 Liver disease - issues with drug metabolism
 CV effects: Arrhythmias (issues in administration of LA containing adrenaline)
 GI system: consider effects of NSAIDs (in pain medication prescription)
 Vitamin deficiencies  poor wound healing

2) Drug dependence
Dependence commonly develops with:
- Sedatives
People likely not to disclose, by - Stimulants (meth mouth) --------->
generally indicated with oral - Hallucinogens (cannabis)
presentation - Narcotics
Dental implication of drug dependence:
 Poor dental attendance (access difficulties)
 Social factors
o Homelessness (oral hygiene difficult)
 Erosion
 Caries - Poor saliva flow (opioids in particular)
- High sugar diet
 Increased risk of endocarditis in injecting drug users
 Drug tolerance
 Head and neck cancer risks (risk factor)
 Drug seeking behavior (be aware, caution of people requesting particular drugs)
Eating disorders
1) Anorexia nervosa Marked weightloss arising from food avoidance, often associated with
binging, purging, excessive exercising, use of diuretics and laxatives
Dental relevance of anorexia nervosa:
Aetiology - Trigger - individuals self perception
 Tooth erosion due to vomiting
from environmental influences
 Low body mass may require
considerations in the dosage of drugs
 Postural hypotension

2) Bulimia nervosa People usually at normal weight, but have a fear of fatness, recurrent
binging and purging

Aetiology: - Unknown (Environmental influencing self perception0


Symptoms: - Russell’s sign from inducing vomiting (sign of contact from incisors on hands)
Dental relevance of bulimia
nervosa:
- Tooth erosion due to vomiting
o Most effected teeth are
the lingual surfaces of
maxillary incisors

ONCOLOGY

…Localized cancer is easier to remove


…Cancer that has metastasized is staged with the TNM method:
T = primary tumor
--describes the size of the primary lesion
N = lymph node involvement
--indicates the advancing lymph node involvement and how far
the cancer has spread
M = metastasis
--indicates the absence or presence of distant metastases

Pathogenesis basics:
Changes to the replicative nature of the cells: cells are programmed to divide, differentiate and die
But in cancers:
- Self-stimulating pro growth signals – over activation of proto-oncogenes
- Insensitivity to anti growth signals (evasion of apoptosis – stimulated by the P53 gene)

Head and neck cancers – 90-95% are Squamous Ceel Carcinomas


|---> The term covers a range of different cancers:
|----> Paranasal sinuses
|----> Nasal cavity
|----> Oral cavity
|----> Pharynx
\----> Larynx
Risk factors:
 Tobacco  Herpes simplex virus – vaccinations for
 Alcohol both males and females is reducing
 Human papilloma virus  Radiation
 Human immunodeficiency virus  Poor oral hygiene

Symptoms:
 General signs:
-> Time present (symptoms can be present for months)
-> Not relapsing (cancers will not wax and wane)
 Symptoms including:
-> Horseness of voice
-> Sore throat
-> Tongue pain (thickening can be palpated)
-> Otalgia – pain in the ear
-> Dysphagia – difficulties in swallowing
-> Cough
-> Bleeding

Oral lesions: premalignant changes:


Leukoplakia – Hyperparakeratosis – epithelial hyperplasia without
dysplastic changes
Erythroplakia – Epithelial dysplasia and is associated with carcinoma in situ
or invasive tumor
Leukoerythroplakia Excessive amounts of keratin production

Prognosis:
- Morbitity and mortality of head and neck caners are high

Treatment:
- Mainly surgical removal
o And/or: Radiotherapy | Chemotherapy
Dental considerations of patients with head and neck cancers:
Prior to Radiotherapy: Patient should be deemed dentally fit
- Ensure pt. can maintain the oral condition throughout their treatment
- If not ----> Extract (tx can commence 7-10 days post extractions)

Extraction in pt. undergoing head and neck radiotherapy:


 Risk of osteoradionecrosis
 More common of mandibular teeth
 Painful/debilitating condition – exposed bone after extraction
 Prophylactic antibiotics – Consider provision (may reduce risk)
 WITH hyperbaric oxygen therapy (AB alone might not be
enough to prevent)
 TX is complex – usually hyperbaric oxygen therapy with surgical
resection
**Specialist review required prior to extraction**

Chemotherapy –
- Often associated with mucositis down the length of the GI tract
o ^^limits the dosage provided
- Chemotherapy results in reduced salivary flow
- Patients to be deemed dentally fit priot to chemotherapy
- Dental treatment should be provided between chemotherapy sessions
o Extraction sockets generally heal well
Question 17: Are you allergic to any tablets, medicines or latex?
ALLERGIES Abnormal, hypersensitive response of the immune system to a substance foreign introduced

Numerous common causes of allergies, most common related to dentistry are:


 Antimicrobials (penicillins)
 Local anaesthetics (true allergies are rare, generally to presevatives)
 Latex
Prior to treatment:
- Determine If the patient has an allergy (Med Hx) - AVOID
- If patient has a know allergy, instruct them to bring their medication

 Involving the humoral or cellular immune system


 Type 1 immediate (humoral) = IgE mediated
 Type 2 (humoral) = IgG & IgM mediated
 Type 3 (humoral) = immune-complex mediated
 Type 4 delayed (cellular) = delayed T-cell mediated response
…Type 1 diabetes / Contact sensitivity (eczema)

Types of allergic reaction:

(1) Urticaria (contact urticaria/ dermatitis):


- Erythematous / oedeomatous swelling of the dermis
o Superficial - Tend to be itchy
o Deep - painful
- Transient – always few minutes to <24 hours

Causes may be: Latex, Antimicrobials (occuring toward the end of the dosages after 4-10 days)
Treatment: ceasing the causative agent

(2) Angioedoema:
- Acute oedema of the subcutaneous tissue (either single or multiple lesions)
o Can be painful/burning (Not itchy)
o Can occur anywhere, commonly the face, around the eye, lips, tongue…
 Dramatic swelling when of the eyelids, lips
 Laryngeal involvement can cause airway obstruction

Causes: possible drug related


Treatment: management same as uriticaria. Systemic corticosteroids if not controlled by
antihistamines

MANAGEMENT OF: URTICARIA / ANGIOEDEMA


MILD
- Cease dental treatment
- Remove the cause / allergen (cease administration)
- Provide oral antihistamine
o During the day: Cetirizine, desloratadine, fexofenadine, loratadine
o Poor ^response^, at night take: Cyproheptadine, dexchlorpheniramine, promethazine)
- Chest tightness – use ventolin inhailer
**Refer patients with extensive symptoms to GP**

ASSOCIATED WITH HYPOTENSION / EVIDENCE OF ANAPHYLAXIS (JUST TREAT AS


ANAPHYLAXIS)
 Treat as anaphylactic reaction – see next page
(3) Anaphylaxis and anaphylactoid reactions: A serious allergic reaction that is rapid in
onset and may cause death

Type I hypersensitivity reaction: ----> Anaphylaxis = IgE mediated


----> Anaphylactoid = Pseudoallergenic

Clinical features:
- With Parenteral / mucosal exposure to cause/drug:
o Symptoms within Minutes
- With ingested cause:
o Symptoms 30min – hours

DENTAL EMERGENCY: ANAPHYLACTIC/ANAPHYLACTOID REACTION


- Cease dental treatment
- Remove the cause / allegen (cease administration)
- Assess the severity of the reaction
- Call 000
- Give intramuscular injection of adrenaline:
o (adult or child) 0.01 mg/kg adrenaline (up to 0.5mg), IM into Anteriorlateral
thigh/tongue/FOM
OR
o 0.3mg adrenaline (Child 10-20kg – 0.15mg), IM via preloaded injector (epipen)
into Anteriorlateral thigh
 REPEAT every 3-5 minutes until response / assistance arrives
- Lie patient flat
- Provide high flow oxygen
- Commence CPR if required
**Patient must go to emergency department

Latex allergies Type: (Type IV) Commonly delayed hypersensitivity response


(contact dermitisis/urticaria– minutes to hours after exposure)
…V. rarely severe (anaphylaxis)
Dentistry implications:
- Use of latex free rubber dam and rubber gloves
Question 19: Are you currently taking any medication, or have you taken any in the last 3 months?

See DEN3ICP PHARMACOLOGY Notes

Question 19: Are you pregnant?

PREGNANCY

Questions to ask?
 How many months pregnant are you?
 Have you been to the doctor?

Dental Significance:
 Defer elective treatment in 1st and 3rd trimester:
o 1st trimester = organogenesis – formation of organs easily effected (17-60 days)
o 3rd trimester = may induce labour
 Mobility issues:
o Adjust chair height
o Pillows if required
o Place patient slightly on their left – increases blood flow to foetus
 LA:
o Articane – least placental barrier permeation (high protein binding)
o Do not use LA containing the vasoconstrictor felypressin (chemically similar to oxytocin
may stimulate uterine contractions)
 Prescription of medications:
o Be aware of teratogenic effects of drugs
o Refer to Therapeutic Guidelines – Drug use in pregnancy and breastfeeding p 210-212
 Oral Manifestations:

Manifestation Pathophysiology Signs/symptoms Management

Pregnancy
 Exaggerated  Begins at  OHI
gingivitis interdental/margina  Debridement
inflammatory response to
Often initiates in l gingiva  Savacol (CH)
local irritants
2nd month of  Fiery red and mouthwash if cannot
  in oestrogen and
pregnancy oedematous gingiva brush
progesterone altering
fibrinolysis and   Tender to palpation
NOTE:  May entail a
capillary dilation (gingival
pregnancy does pyogenic
inflammation)
not induce PPD, granuloma/
but may  Sub-par OHI due to
morning sickness and pregnancy tumour
modify/worsen (see below)
what is already feeling nausea whilst
present. brushing
 Commonly on  OHI
interdental papilla  Localised
 Benign hyperplastic
Pyogenic  Localised debridement
growth in a localised area
granuloma  Inflamed swelling  May excise if doesn’t
 See ABOVE 
 Commonly at the solve
end of 1st trimester
 Mobility is a sign of  Pregnant pt  OHI
gingival disease, complaining of  Use of multivitamins,
disturbance in attachment tooth mobility especially vitamin C
apparatus, and mineral  Successful delivery of
changes in the lamina NOTE: must assess if newborn
Tooth mobility dura there is underlying
 Vitamin deficiencies reasons for tooth
contribute to this mobility that are
 Calcium is readily pathological.
mobilised from bone to
supply fetal demands
 Non-Carious irreversible  Rounding of sharp  Rinse after
tooth loss due to chemical angles regurgitation with a
dissolution by acids  Dentine cupping or neutralising solution
 Hypersensitive gag reflex scooping e.g. baking soda,
in combination with  Thinned enamel water
Dental erosion
morning sickness  Restorations may  pH neutralizing MW
contributing to appear above the such as sodium
regurgitation (causing tooth surface bicarbonate (Peter
halitosis) Mac MW) or sugar-
free chewing gum
OCCULAR EMERGENCIES …

Prevention:
- Use of safety glasses (esp. when using rotary instruments or chemicals)
o By both pt. and dental team

Sources of injury:
(1) Chemicals (e.g. endodontic irrigating)
DENTAL EMERGENCY: CHEMICAL EYE INJURY
- Cease dental treatment
- Immediately irrigate with copious amounts of water
o Holding eyelids open
o Remove contact lenses
o Continue irrigation for min. 15minutes (poured from cup/tap)
 X not with eyecup – need water moving X

o Weak chemical injury: organize medical review that day


o Caustic chemical injury / marked inflammation
 Dial 000
 Continue irrigation until assistance arrives

(2) Foreign bodies (fragments of fillings / calculus – may be lodged on the eye)
DENTAL EMERGENCY: FOREIGN BODY LODGED ON THE EYE
- Cease dental treatment
- Immediately irrigate with copious amounts of water
o Holding eyelids open
o Do not touch eye surface, do not attempt to remove with anything other than water
- Check after 5 minutes
o Continue until 15 minutes if not removed
- Gone  organize prompt medical review
- Still present  transfer to emergency

(3) Penetrating objects (drills, endodontic instruments)


DENTAL EMERGENCY: PENETRATING OBJECTS
- Cease dental treatment
- Dial 000
o Prevent the pt. from rubbing the eye
o Cover the eye with eyeshield / polystireen cup and tape
o Keep patient calm until assistance arrives
 DO NOT attempt to pull out the object
 DO NOT irrigate the eye
BASIC LIFE SUPPORT:

CPR = 30 compressions for 2 breaths

S-ar putea să vă placă și